Vous êtes sur la page 1sur 136

Laws of Motion and Friction

SECTION A
Constraint Equation
1. In the figure, ends P and Q of an inextensible string move down-wards with
uniform speed u. Pulleys are fixed and mass less. What is the speed v of the
mass m in upward direction?

x2 c1 2 y 2 d 2 c2 x1 cons tan t
Sol:-

dx2
1
dx
02
2y 0 1 0
2
2
dt
dt
2 y d

dx2 dx1

dt
dt

dy

0 u u 2(cos )v 0
dt
y 2 d 2

2u 2(cos )v 0
v

u
cos

u v cos 0
[ As

dx1
dx
dy
u
v p , 2 vQ , vm ] v
dt
dt
dt
cos

2.In the figure m 2 > m1 pulley and string are light. Using constraint method find the
relation between accelerations of m1 and m2.

Solution:x1 +c+x2 = constant

dx1
dx
0 2 0
dt
dt

dx1 dx2
d 2 x1 d 2 x2

0 2 2 0
dt
dt
dt
dt

a1 a2 0 a1 | a2 | a 2
3. The ring m2 and block m1, are held in the position shown in figure and the system
is released. If

m2 > m the ring m2 slides down along the smooth fixed vertical rod, find

Solution

The length of the string is constant,

x c y 2 d 2 cons tan t

dx
1
dy
0
2y 0 0
2
2
dt
dt
2 y d

As

dx
dy
uand
v
dt
dt

u v cos 0

u cos .v 0
v

u
cos

v
u

u
cos

v
1

u cos

4. Find the constraint equation between the accelerations of the blocks M 1, m2 and
m3.
Solution:-

For string 1: X1 + C+ XB = constant

d 2 x1
d 2 xB

0
dt 2
dt 2

a1 aB 0 . 1

For string2: X2 - XB + C1 + X3 - X = constant


= X2 + X3 -2 XB + C1 = constant

d 2 x2 d 2 x3 d 2 xB
2 2 2 0
dt
dt
dt

a2 a3 2aB 0 . 2

From Eqs. (1) and (2), we get:


=a2+a3-2(a1)=0
=>2a1+a2+a3=0
5. Assuming pulleys and strings are light, find constraint relation between the
accelerations of the bar m1 and the block m2 as in the given figure.

Solution
For 1,

X1 + XB = constant

d 2 x1 d 2 xB
2 0
dt 2
dt

a1 aB 0 . 1

For 2,
(x1 xB)+(xc -xB) constant
=>x1 2XB +Xc =constant
For 3,
(x1 - xc) + (x2 - xc) = constant
=> x1 + x2 - 2xc = constant
=> a1 + a2 - 2ac = 0 ... (3)
From Eqs. (1) and (2)
a1-2(a1)+a=0
=> a1+2a1+a c =O
=> 3a1+a c =0
a-3a1
From Eq. (3)
a1+a2-2(-3a1)=0
=> a1+a2+6a1=0
=> 7a1+a2=0
6.A block is pulled on a smooth surface with the help of a rope and rope is pulled
with speed u as in figure. Find the horizontal velocity of the block. (Assume the
contact doesn't lose during motion.)

Solution

x c y 2 d 2 cons tan t

dx d y 2 d 2

0
dt
dt
dx d y 2 d 2

0
dt
dt

dy
0
2 y d dt

u sin ( v) 0 as

sin ( v) u

v
cos ec
u

dy
v
dt

=>
7. Using the constraint equation find the acceleration of the if the acceleration of
the block m1 is a as in figure.

Solution
For l,a1=aA ...(l)
For 2, XA + (xA - x2) = constant

=> 2xA - x2 = constant

d x A d x2
2 0
dt 2
dt

2a A a2 0

2a1 a2 0 2a1 a 2
.. acceleration of block m2 = 2a.
8. In the figure, assuming pulleys and string are light, if the directions of
accelerations are given then find the constraint equation.

Solution

x1 x2 2 x3 cons tan t
d 2 x1 d 2 x2 d 2 x3
2 2 0
dt 2
dt
dt
Since x1 and x2 are assumed to be decreasing with time,

d 2 x1
d 2 x2

a
and
a2
1
dt 2
dt 2

therefore,
and x3 is assumed to be increasing with time, therefore,

d 2 x3
a3
dt 2
Thus a1 a + 2a3 = 0 or a1 + a2 = 2a3.
9. Assuming pulleys and string are light, find the constraint equation for
accelerations of m1 and m2.

Solution
6x1 + c + 5x2 + c' = constant

d 2 x1
d 2 x2

5
0
dt 2
dt 2

6a1+5a2=O
10. Assuming pulleys and string are light, find the constraint equation for
accelerations of m1, m2 and m3.

Solution
x1 + x2 +X = constant

d 2 x1 d 2 x2 d 2 x3
2 2 0 a1 A a2 a3 0
dt 2
dt
dt
11. An inextensible string AB is tied to a block B of negligible dimensions and passes
over a small pulley C so that the free end A hangsh1 unit above the ground on
which the block B rests. In this initial position shown in figure, the free end A is h
unit below C. If now the end A moves horizontally with a velocity u, obtain an
expression for the velocity of the block at any time t.

Solution
In time t, the end A moves to the position A1. So that AA 1 = Ut . The block B moves
upwards to the position B1. Let BB1=y.

In the ACA1 h y

h 2 ut

Then length A1 C = h + y
Or h2+y2+2hy=h2+u2t2
y2+2hy-u2t2=0.(1)
After solving equation (1), we get

y h h 2 u 2t 2 ... 2
This is the equation for the displacement y of the block. Velocity of the b k:loc

dy d
h (h 2 +u 2t 2 )1/2
dt dt

1
(h 2 +u 2t 2 )1/2 2u 2t
2

u 2t
v 2
(h +u 2t 2 )1/2
Or
12. A rod of length / is inclined at an angle with the floor against a smooth vertical
wall. If the end A moves with velocity v1 and the rod makes an angle
horizontal, what is the velocity of end B at that instant?

Solution

with

x 2 y 2 l constant
=> x2 +y2 = l2 = constant

2 x

dx
dy
dx
dy
2y
0 x y
0
dt
dt
dt
dt
dy
v2
dt

xv1 yv2 0 As

xv1 yv2

v1 y
v
1 tan v2 v1 cot
v2 x
v2

13. As shown in figure, the velocity of rod at any instant in downward direction is u.
Then what will be the velocity of triangular wedge in horizontal direction at that
instant?

Solution:-

tan

y
dy
dx
y x tan
tan
x
dt
dt

u v tan v

u
v u cot
tan

14. A racing car travelling along a track at a constant speed of 40 m/sec. A


cameraman is recording the event from a distance 40 m directly away from the
track as in figure. In order to keep the car under view, with what angular velocity
the camera should be rotated after 1 sec from the start?

Solution

Angular velocity
tan
At t=1sec

d
?
dt
x
x 40 tan
40

dx
dx d(40 tan )
dx
d
d
dt

40sec 2

dt
dt
dt
dt
dt 40sec 2

d
40
d
1
dx

as v 40m / s
2
2
dt
40 sec
dt sec
dt

At t= 1 sec, x=40m and d=40m

tan

x
40

1 .. 450
40
40

d
1
1

rad / sec.
2
0
dt sec 45
2
Laws of Motion and Friction
I5. Two unequal masses moving along straight lines are brought to rest by equal
resistance forces. If one mass moves twice as long (time) as the other but goes only
1/3 of the distance covered by the other before coming to rest. Find: (i) the ratio of
their velocities; and (ii) the ratio of their masses.
Solution

v1
t1
a1
(A) F = m1a1 and

As

t2
2
t1

t2 a1
. 2..........(1)
t1 a2
AgainS1

v12
v2
and S 2 2
2a1
2 a2

S1 v2 2 a1 1

. .......(2)
S2 v12 a2 3
From Esq. (1) and (2),

v2
t2
a2
(B) F = m2a2 and

v2 2 1 v2 1 v1 v1
. , , or 6
v12 3 v1 3 v2 v2
a1
F F
12.So m1 : m2 : a2 : a1 1:12.
a2
a1 a2
16. Calculate the forces of couplings in the figure.

Solution

Acceleration of the system

F
110
110
=

2 m / sec 2
m Totalmass
55

T1=30X2=60 N; T2=15X2=30 N; T3=5X2=1O N.


17. The pulley arrangements of Figure (a) and (b) are identical. The mass of the
rope is negligible. In figure (a), the mass m is lifted by attaching a mass 2m to the
other end of the rope. In Figure (b), m is lifted up by pulling the other end of the
rope with a constant downward force F = 2mg. The acceleration of m is the same in
both cases. Is it correct?

Solution
Let a be the acceleration in the case in Figure (a).
T mg=ma
.(1)
2mgT=2ma (2)
Adding Esq. (1) and (2),
Mg= 3ma or
a=g/3
Let a' be the acceleration in the case in Figure (b).
T - 2 mg = 0 or
T = 2mg
and T - mg = ma'
or 2mg - mg = ma'
or mg = ma' i.e., a' = g
Acceleration in case (b) is greater than that
in Case (a). So, the statement is incorrect.

18. Two masses m and 2m are connected by a mass less string which passes over a
pulley as shown in the figure. The masses are initially held with equal lengths on
either side of the pulley. Find the velocities of the masses at the instant the lighter
mass moves up a distance of 6.54 m. The string is suddenly cut at that instant.
Calculate the time taken by each mass to reach the ground. (g = 9.81 m/s 2)

Solution

Acceleration of the system

(2m m) g g
3.27 m / sec 2
2m m
3

2m moves down and m moves up with acceleration 3.27 m/s 2 from rest.
If v is velocity after moving 6.54 m,
v2=2aS=2X3.27X6.54
..V=6.54m/s
Initially, both are at height of 13.08 m from the ground. When they moved 6.54 m,
height of m from ground = 13.08 + 6.54 = 19.62 m and height of 2 m from ground
= 13.08 - 6.54 = 6.54 m. When the string is cut, m has upward velocity 6.54 m/s
and 2 m, the same downward velocity.

Motion of m: It takes

6.54 2

9.81 3

to go to the highest point. In this time it goes up

u
6.54 6.54

2.18m
2g
2 9.81
2

total distance it falls 2.18 + 19.62 = 21.8 m.

2h
2 21.8

2.11s
g
9.81
Time to reach ground =

2
3

Total time to reach ground = 2.1 +


= 2.78s
Motion of 2m: It descends with u = 6.54 m/s.

1
S ut at 2
2

6.54 6.54t 2

3 2
t 2t 2 O
2

3t 2 4t 4 0

1
9.81t 2
2

4 16 48 4 8 4 2

s 0.67 s
6
6
6 3

19. Find the tension of strings connecting the masses shown in the figure. The
pulleys are mass less and the surface friction is zero. Find the relation between the
masses m, m' and M, if M is in eqilibriurn.

Solution
Tension is the same throughout. If M moves down, m moves to the right and m' up.
If m moves to the right by x, m' moves up by y,

M moves down by

aa'
2

.So, the acceleration of M, m, and m' are, respectively,

a and a.

So, T=ma,Mg-2T=

a a'
2g

x y
2

M ( a a ')
2

,T-m' g=m 'a'

2Mg 4T
4T T T m ' g
2g

M
M m
m'

4T T T
3g
4 1 1

g ;3 g T
;T
4 1 1
M m m'
M m m '

M m m'

If M is in equilibrium, 2T = Mg

Mg
3g
M M
1 1

;4

6 ' orM 2
4
1
1
2
m m'
m m '

M m m'

2mm '
m m'

Thus, M is the harmonic mean of m and m'.


20. Find the acceleration of the system shown in the figure. The pulleys are smooth
and the threads are of negligible masses.

Solution

The tension

2m1m2
effective acceleration
m1 m2

T1

264
(g a) 4.8(g a)
64

T2

2 9 1
(g a) 4.8(g a)
9 1

also, T=2T1 +2(ga)=2X4.8(ga)+2(ga)= 11.6(ga)


and T=2T+2(g +a)2X 1.8(g+ a)+2(g+ a)5.6(g + a)
11.6(ga)=5.6(g +a)

6 g 6 9.8 588

3.4m / s 2
17.2 17.2 172

4 times
21. In the arrangement shown in the figure, the mass of body 1 is
0 that
of body 2. The height h = 20 cm. The masses of the pulleys and the threads as well
as the friction are negligible. At a certain moment, body 2 is released and the
arrangement is set in motion. What is the maximum height that body 2 will go up
to?

Solution
If mass 1, moves down 1 cm, mass 2 moves up 2 cm.
So, its acceleration will be twice that of 1.

mg-2T=

ma

Tmgm2a

2T-2mg4ma

mg-2mg=(

+4)ma

or

( 2) g
4

a=

2( 2) g
4

acceleration of body
With this acceleration, it rises to a height 2h. Velocity on reaching 2h is given by

v 2 2as

2 2 g ( 2)
2h
4

On reaching this height, it rises again until its velocity is zero.

The further height reached h '

Total height reached 2h

2 2 g 2 2h
v2
4( 2)

h
2g
( 4)
2g
4

4 2 4
4( 2)
2( 2)
2h 1
= 2 h

( 4)
( 4)
4

6 h
4
22. A trolley of mass 100 kg, starting from rest, describes 100 m in 10 second. At
that instant, i.e., at the commencement of the 111 second, two packets, each of
mass 12.5 kg, are gently placed in the trolley. How far does it move in the next 10
seconds assuming that the forces on the trolley remain the same throughout?
Solution

1
Sinces ut at 2
2

1
100 0 a100
2

.. a 2 m / sec 2

The velocity at the end of 10 seconds = v = u + at = 0 + 2 X 10 = 20 m/s


F ma= 100X2200N.

200
125

During the next 10 seconds, acceleration =


= 1.6 m/sec2
125
The velocity at the commencement of the 11th second v' is given by
mu = m' v' (law of conservation of momentum)
100 X 20 = 125 X v'
100 X 20

V =

100 20
16m / sec
125

The distance travelled in the next 10 seconds


16 10

1
Ut at 2
2

1
1.6 100 240m.
2

23. Two blocks of mass m = 5 kg and M = 10 kg are connected by a light string


passing over a pulley B as shown. Another light string connects the centre of pulley
B to the floor and passes over another pulley A as shown. An upward force F is
applied at the centre of pulley A. Both the pulleys are mass less. Find the
acceleration of block m and M, if F is:
(a) 500 N
(b) 300 N
(c) 100 N (g = 10 m/s2)

Solution

Let T0= tension in the string passing over A


T = tension in the string passing over B
2T0=F and 2T=T0

=> T=F/4
(a) T=F/4125N
As T> mg and Mg, both the blocks will accelerate upwards.

Acceleration of m, a1
Acceleration of M , a2

T Mg
125 100

15 m / s 2
M
10
T Mg
125 100

2.5 m / s 2
M
10

M 10
(b) T=F/4=75N
As T < Mg and T> mg, M will remain stationary on the floor, where as m will move.

Acceleration of m, a1

T Mg
75 50

5 m / s2
M
5

(c) T=F/4=25N
weights of blocks are
mg = 50 N Mg= 100 N
As T < mg and Mg both, the blocks will remain stationary on the floor.
24. In the arrangement shown, the blocks A, B, C and D have masses m 1, m2, m3
and m4 respectively. The springs are weightless and have force constant k and the
string and pulley are light and smooth. The system is maintained in equilibrium by
the thread DG connecting block D to the ground. If the thread is cut at a certain
moment, determine the accelerations of the blocks immediately afterwards.

Solution

Considering the equilibrium of blocks


T1T4m1g
...(1)
T1=m2g
(2)
T2-T3=m3g
(3)
T3-T4=m4g
(4)
From (1) and (2), T2 = (m1 + m2)g
From (3) and (4), T2 - T4 = (m3 + M4)g
T4 = (m1 + m2)g - (m3 + m4)g = (m1 + m3 - m4)g
When the thread DG is cut, an unbalanced force equal to T 4 acts immediate on D in
the upward direction.

Initial acceleration of D a4

m m2 m3 m4 g
T4
1
m4
m4

25. Both the blocks are resting on a horizontal floor and the pulley is held such that
string remains just taut. At moment t = 0, a force F = 20t N starts acting on the
pulley along vertically upward direction as shown in figure. Calculate:
(a) velocity of A when B loses contact with the floor.
(b) height raised by the pulley up to that instant. (g = 10 m/s 2)

Solution
(a) Let T be the tension in the string. Then, 2T = 20 t or T = 10t N
Let the block A loses its contact with the floor at time t = t 1 This happens when the
tension in string becomes equal to the weight of A. Thus,
T= mg
or
10t1 = 1 X 10
or
t, = 1s . . .(i)
Similarly, for block B, we have
10t2 = 2 X 10
or
t2 = 2s
. . .(ii)
i.e., the block B loses contact after 2s. For block A, at time t such that t
its acceleration in upward direction. Then,
10t - 1 X 10 = 1 X a = (dv/dt)
or
dv = 10 (t - 1) dt . . .(iii)
Integrating this expression, we get
v

dv 10 t 1 dt or

v 52 10t 5 ... iv

Substituting t = t2 = 2 s

or

v = 20 - 20 + 5 = 5 m/s . . .(v)

t1 let a be

(b) From Eq. (iv), dy = (5t2 - 10t + 5) dt

. . .(vi)

Where is the vertical displacement of block A at time t ( t 1).


Integrating, we have
y h

t 2

t 3
dy t1 (5t 10t 5)dt h 5 3
y 0
2

10

t2

Height raised by pulley up to that instant

5
2
5 t 1 m
3
1

h 5
m
2 6

26. A cat of mass m = 1 kg climbs to a rope hung over a light frictionless pulley. The
opposite end of the rope is tied to a weight of mass M = 2 m lying on a smooth
horizontal plane. What is the tension of the rope when the cat moves upwards with
an acceleration a = 2 m/s2 relative to the rope?

Solution
Let a be the absolute upward acceleration of the monkey and a' be the absolute
downward acceleration of the rope. a' is also the rightward acceleration of M. Then,
b = a - (a') (since relative acceleration is the vector difference between the
absolute accelerations) or b - a = a' Considering upward motion of the cat
T - mg
= ma ... (i)
Considering rightward motion of M
T = Ma' = M(b - a) ... (ii)

mM
m 2m
2m
12 8n
g b
(10 2)
mM
3
m 2m

From (i) and (ii), we get


27. Blocks A and C start from rest and move to the right with acceleration 4 = 6t
m/s2 and ac = 3 m/s. After what time does the block B again come to rest?

Solution
From constraint relations we can see that acceleration of block is

dvB
3 6t )
1.5 3t
1.5 3tor
2
dt

aB

vB

or dvB (1.5 3t )dtor ,vB 1.5 t 1.52 tor vB 0att 1s


7

25

sin 1

28. A tram moving up an incline of


goes with an acceleration of m/s2. What
will be the 25
acceleration of the same train on a level road if it exerts the same traction force?
The coefficient of friction between the road and wheels is the same, 0.2, on both
sides.
Solution

Traction force mg sin mg cos ma


mg

7
24 mg
84
7 24 1
0.2mg

mg

mg
25
25 5
125
25 125 5

Traction force in the second case =

84mg
84mg 1
84mg
mg
mg
125
125 5
125

F 84 g

m 125

The acceleration =
29. A mass m slides down a smooth inclined plane of inclination a and draws
another mass from rest through a distance d meter in t second along a smooth
horizontal table which is on level with the top of the plane over which the string

gt 2 sin 2d

2d

m
passes. Prove that the mass on the table is
sin a

Solution

m, mg sin T ma
Let the acceleration of the system be a. Then for mass

m, mg sin T ma
and
The mass m' moves a distance d in t seconds

1 2
at
2

2d
t2

2d mg sin

t2
m m'

2md 2m ' d mg sin t 2 2m ' d mg sin t 2 2md m[ g sin t 2 2d ]


gt 2 sin 2d
gt 2 sin 2d

1
2d
2d

m ' m

30 . The system of bodies in the figure starts from rest. Determine the acceleration
of the body B and the tension in the string supporting the body A given the weight
of A = 500 N and of B = 750 N. The coefficient of friction between the surfaces is
0.2.

Solution
Applying Newton's second law of motion to A,

500 T

500
a...(i)
g

Similarly, applying it to B

2T 750sin 750 cos

750
...(ii)
g 2

since if A moves x down, B moves up the plane x/2.


from equation (i) and (ii), we get

430 g
3.06m / sec 2
1375

Acceleration ofB

T 344 N

a
1.53m / sec 2
2

31. In the system of connected bodies in the adjoining figure, the coefficient of
kinetic friction is 0.20 under bodies B and C. Determine the acceleration of each
body and the tension in the chord supporting A.

Solution
Suppose, B moves up a distance x and moves down by a distance y. So, C moves

(2 x y )
down a distance

on its own plane.

a A , aB (2aB a A ),
So, the acceleration of A, B, C are
Applying Newton's second law to A, B, C.

respectively.

T 800 g sin 800 g cos 800aB


where T is the tension of the string.

Or

3
1
4
2T 800 g 800 g 800 aB
5
5
5

2T 480 g 128 g 800 g 2T 608 g 800aB


(1)
Similarly,

400 g T 400a A .....(2)

800 g 2T 800a A
or

Adding Eqs. (1) and (2),

800 g 608 g 800( a A aB )


Now, for C

1000 g sin 1000 g cos T 1000aC 1000(2aB a A )


3 1000
4
or1000 g
g T 1000(2aB a A )
5
5
5
or 600 g 160 g T 1000(2aB a A ) 440 g T 1000( 2aB a A )
600g 160g T= 1000

T 304 g 400 aB from 1


(2aBand T

136 g
2.4aB a A ......(4)
1000
From (3) and (4) ,

136 g 2400aB 1000 a A


1000

3.4aB

136 g 240 376 g

1000 1000 1000

aB

376 g
1.084m / sec 2
1000 3.4

136 g
136 9.8
2.4 1.084
2.6016 1.3328 1.27 m / sec
1000
1000 3.4
T 400( g a A ) 400(9.8 1.27) 400(8.53) 3412 N
a A 2.4aB

1/ 2.
32. In the figure, coefficient of friction between the two blocks is given as
Find the force of friction acting between the two blocks. (g=10m/s 2)

Solution
Let acceleration of both the blocks towards left be a.

a ( f 2) / 2 (20 f )or , 2 f 4 20 forf 8 N


Then
Maximum friction between the two blocks can be

f max mg (m 2kg ) (0.5)(2)(10) 10 N


f f max
Now since
Therefore, friction force between the two blocks is 8 N.

33. Two blocks A and B of mass 2 kg and 4 kg are placed one over the other as
shown in figure. A time varying horizontal force F = 2t is applied on the upper block.
(t is in second and F is in newton) Coefficient of friction

between A and B is
smooth (g = 10 m/s2)

1
2

and the horizontal surface over which B is placed is

Solution
Limiting friction between A and B is

1
f L mA g (2)(10) 10 N
2
Block moves due to friction only . Therefore, maximum acceleration of can be

amax

f L 10

2.5m/ s 2
mB 4

Thus both the blocks move together with same acceleration till the common
acceleration becomes

2.5 m/ s 2
after that acceleration of will become constant while that of A will go on
increasing To find the time when thew acceleration of both the blocks becomes

2.5 m/ s 2
(or when slipping will start between A and ) we will write

2.5

F
2t
t 7.5s
(mA mB ) 6

Hence , for

t 7.5s

a A aB

F
2t t

(mA mB ) 6 3

t 7.5saB 2.5 m/ s 2
For

Constant

aA

F fL
mA

aA

2t 10
2

aA t 5

And
or
or
34. There blocks A, and C of equal mass m are placed one over the other on a
Smooth horizontal ground as in the figure Coefficient of friction between any two
blocks of A, and C is Find the maximum value of mass of blocks D so that the
blocks A, and C move without slipping over each other .

Solution
Blocks A and C both move due to friction But less friction is available to A compared
with C because normal reaction between A and is less Maximum friction between
A and can be

f max mA g (1/ 2) mg
amax
Maximum acceleration of A can be

amax

f max g

m
2

mD g
g
mD g

mD 3m
3m mD
2 3m mD

Further
35. Figure shows a small block A of mass m kept at the left end of a plank B of mass
M = 2m and length l. The system can slide on a horizontal road. The system is
started towards right with the initial velocity v The friction coefficients between the
road and the plank B IN is 1/2 and that between the plank and the block is 1/4. Find:
(a) the time elapsed before the block separates from the plank.
(b) displacement of block and plank relative to ground till that moment.

Solution
There will be relative motion between block and plank and plank and road. So at
each surface limiting friction will act. The direction of friction forces at different
surfaces are as shown in figure Here,

f1

mg
4

and

3mg
1
f1 (m 2m) g
2
2

Retardation of A is a1 =

f1 g

m 4

and retardation of B is a2 =
Since, a2 > a1

f 2 f1 5
g
2m 8

Relative acceleration of A with respect to B is a r = a2 - a1 = g


Initial velocity of both A and B is v. So, there is no relative initial velocity. Hence,
(a) Applying

1
s at 2
2

1
3
l a, t 2 gt 2
2
16

=>
(b) Displacement of block

1
S A u A t a At 2
2
S A 4v

S A 4v
=>

l
3g

t=

l
1 g 16l

3 g 2 4 3 g

g
a A a1
4

l
2
l,
3g 3

or

Displacement of plank SB = U Bt -

S B 4v

l
1 5
g
3g 2 8

S B 4v

l
5
l,
3g 3

or

16l

3 g

1
2

aBt2

a B a2

5
g
8

or
36. The coefficient of friction between the block A of mass m and block B of mass

1
3

2m is

.The inclined plane is smooth. If the system of blocks A and B is

released from rest and there is no slipping between A and B, then


value of

Find the

Solution
When there is no slipping, then both the blocks move together with acceleration a =
g sin

down the plane. Horizontal component of this acceleration is a H = a cos

and vertical component is av = a sin


cos

And av = a sin

= g sin

, where a = g sin

aH = a cos

= g sin2 .

N = normal reaction between A and B.


Equations of motion in horizontal and vertical directions give:
mg-N=mav or N = mg - mav = mg - mg sin2

f = friction force p N

or

mg cos

maH

- mg sin

cos

=>

tan

= Mg cos2

or

-1

tan (

=>
tan-1
=>
.. =6
37. In the arrangement shown in the figure, M A = MB = 1 kg. String and pulley are
mass less. Block B is resting on a smooth horizontal surface, while friction
coefficient between blocks A and B is p = 0.4. Find the maximum horizontal force
that can be applied so that block A does not slip over the block B.

Solution
Net horizontal force on block B is zero. Hence, the given figure (a) can be replaced
by figure (b).

f max mA g
Maximum value of friction is
(0.4) (1) (10) = 4 N. Block B moves due
to friction. Therefore, maximum common acceleration of the two blocks can be

amax

f max 4
4m / s 2
mB 1

Fmax ( mA mB )amax (1 1)(4) 8 N

and
38. Find the maximum value of mass of block C of that neither A nor B moves. (m A =
100 kg, mB = 200 kg, Pulley and strings are mass less. Coefficient between A and B

and between B and horizontal surface is

= 0.4)

Solution
Maximum friction that can be obtained between A and B is

f1 mA g
= (0.4) (100) (10) = 400 N
and maximum friction between B and ground is

f2 = (m A + m B)g (0.4) (100 + 200) (10) = 1200 N


Drawing diagrams of A, B and C in limiting case,

Equilibrium of A gives
T1= = 400 N ... (i)
Equilibrium of B gives
2T1 +f1 +f1 = T2 =>
T2 = 2(400)+ 400 + 1200 = 2000N
...(ii)
and equilibrium of C gives
mCg =T2
=> 10 mc = 2000 ..
MC = 200 kg = 2 X 102 kg
39. An open car of mass m0 is running on smooth horizontal rails under rain falling
vertically which it

catches and retains in it. If v0 is the initial velocity of the car and
water falling into
the car per second, find the distance travelled by it in time t.
Solution:

dv
dm
v
0
dt
dt

(m0 t )
So,

ln

v v0

t) + C

So, C = In v0 + In m0

m0
m0 t

=ln(m0+

dv
dt

dt m0 t

Integrating, In v = - In (m0 +
0, v = v0

v
v0

dv
v 0
dt

t)+ln m0=ln

m0
ds

m0 t dt

the mass of

dt
ds

m0 t m0 v0
Integrating, we get

because when t =

1
s
In(m0 t )
D

m0 v0

when t=0,s=0, ..

In m0 =D ..

1 (m0 t )
s
In

m0
m0 v0

s
m0 v0

In (m0+

t)=

ln m0

m0 v0
t
In 1

m0

40. A rain drop of mass m starts falling from rest and it collects water vapors and
grows. If it gain
Solution

kg/s, find its velocity at any instant.

d
( mv) mg;
dt

As

dm

dt

or

When t=0, m=m0 ..m=m0+ t

Integrating, (m +
When t=0,v=0

m0

So,

t 2
t gdt m0t g C
2

, where C is a constant.2 )

m0t

where K is a constant.

d
m0 t v m0 t g
dt

t)v =

.. C=0 ..(mo +

m= t+ K

t 2
g
2

t) v=

t 2
t 2
g m0t g t

2m0
2
v

t
m0 t
1
m0
This means velocity changes with respect to time.
41. A clean body of mass 100 g starts with a velocity of 2 m/s on a smooth
horizontal plane, accumulating dust at the rate of 5 g/s. Find the velocity at the end
of 20 seconds and the distance travelled during
Solution

d
(mv ) 0
dt

dv
dm
v
0
dt
dt

Here
so
After t seconds, mass of the body = 100 + 5t.

dv
dt

So (100+5t)

+5v=0
v

[Inv]

200

Integrating,

In

v
200

or

dv
5dt

v
100 5t

ln 1005t

20
0

100
200

v
200

1
2

=-ln200+ln100=ln
...
=
=>v=100cm/sec.=1m/sec.
Also, ln v= - ln (100 + 5t)+ In C
When t = 0, v=200cm/sec.
In200-ln100+lnC
:.ln C=ln200+ln1OO=ln(200X100).
In v=-ln(100+5t)+ln(200X 100)

200 100 ds

100 5t
dt

ds s

ds=

200 100dt
100 5t

cm=

200dt
100 5t

20

20
200dt
200

40
40 In 40 In2
[I
n
(100

5
t
)]
0 100 5t
0
100

= 40 X 0.3010X2.303 = 27.7 m
42. A flat car of mass m0 starts moving to the right due to a constant horizontal
force F. Sand spills on
the flat car from a stationary hopper. The velocity of loading is constant and equal

to
kg/s. Find the time dependence of the velocity and the acceleration of the flat
car in the process of loading. The friction is negligibly small.
Solution

dv
dm
v
F
dt
dt

(m0+

t)

dv
dt

Mass of the car at any instant = m0 +

dv
dt

F v m0 t

+v

=F

Integrating, we have in (F -

v) = - in (m0 +

t) + C

F v
dt

dv
m0 t
Whent=0,v=0,

... ln F=-In m0+C

v
t
a c
b a d c
Ft

sin ceif , then

F m0 t
b d
b
d
m0 t

Acceleration =

(m0 t ) F Ft Fm0 Ft Ft
dv d
Ft


dt dt m0 t
(m0 t )2
(m0 t )2
Fm0

t
m0 1
m0

m0 1
m

=
43. A particle moves in a straight line under the action of a retarding force. If its
initial speed is u a
the retarding force is mKv3 where m is its mass, v is velocity at any instant and K is
a constant

1 1

v u

show that
= Kx, x being the distance covered before its speed is reduced to v.
Show t
the time taken to travel is given by
Solution

dv
mKv 3
dt

Integrating

Or

1 1
2 2
u v

m
Also m

or

1
2v 2

=>

1
v

dv
Kdt
v3

=-Kt+ C.

When t=0,v=u

= - 2Kt

vdv

dx

Integrating,

dv
Kv 3
dt

=>2Kt =

=-mKv3

...

dv
v2

:.

1
1
2
2
v
u

=K dx

= Kx + C

When x=0,v=u

..

1 1
Kx
u v

or

1 1
Kx
v u

1
2v 2

=-Kt

1
2u 2

But 2Kt=

1 1
2 2
v u

1 1 1 1 1
2


2
v u 2K v u

1 1 1
2
1

Kx
Kx

v u 2 K
u
2K

x Kx 2

u
2

44. Having gone through a plank of thickness h, a bullet changed its velocity from v 0
to v. Find
time of motion of the bullet in the plank, assuming the resistive force to be
proportional to square of the velocity.
Solution

dv
dt

Resistance =m

Integrating,

1
v

=-Kv2

k
m

or

dv k
dt
v2 m

t +c. When t=0,v=v0

1
Kt 1

v
m v0

1
v0

So,

=c

1 1
Kt
....(1)
v0 v
m
or

Also m

dv
dt

=mv

dv
dx

=-Kv2

Integrating, In v=-

or ln v= -

k
m
t

From (1),

k
m

or

x+D.

dv
Kdx

v
m

When v=v0,x=0 .. ln v0=D

v
v0
x+ln v0

...

In

v0 v
m 1 1
m

k v v0
K
v0 v

(v0 v) h
v0 v log v0
v

kx

m

kx
m

From (2), h

..(2)

m
k

= log

v0
v

t=
45. A motor boat of mass m moves in a lake with a velocity v 0. At the moment t = 0,
the engine is shut down. Assuming the resistance of water to be proportional to the
velocity of the boat, F = - rv (r is a positive constant), find:
(a) how long the boat moved with the shut down engine,

(b) the velocity of the boat as a function of the distance covered till the complete
stop, and
(c) the mean velocity of the boat over the time interval (beginning with the
moment, t = 0) during

which the velocity decreases


Solution

Acceleration of the boat =

Integrating, In v=

In v=

v
v0

rv dv

m dt

dv
r
dt
v
m

t+c When t=0,v=v0 So, C=lnv0

v
v0

r
m

r
m

times.

t+lnv0

or

r
t
m

ln

r
m

r
t
m

or
=
or
v=v0
m i.e., v ->0 when t ->
So; it stops when t is infinite only.

vdv
rv

dx
m

Also

dv
r

dx
m

or

V=

r
m

x+D

r
m

When x=0,v=v0
.. v0 =D
Hence v=
x+v0 v=v0
Total distance it moves before it comes to stop is given by

V=V0

rx
m

=0

x=

rx
m

v0 m
r

we have to find the distance x' covered before the velocity becomes Because

v=v0

x=

rx
m

v0

m 1

v0
r

=v0

rx '
m

or

rx '
m

v0

=v0 -

=v0

1
1

e
But v

rt

=v0

=>

rt
v0
em
v0 /

rt
v0
em
v

rt
Ine
m

t=

rt

em
or

mIne
r
m 1

v0
x' r
( 1)

mIne
t
Ine
r

distance travelled
time

Average velocity =
=
46. A water pipe has an internal diameter of 10 cm. Water flows through it at the
rate of 20 rn/sec. The water jet strikes normally on a wall and falls dead. Find the
force on the wall.
Solution

Mass of water flowing through the tube per second = Av


where A = area of cross section and v = velocity of water of density

v = Av2

Momentum change/second = Av

100

Thus, the force on the wall = Av


=
x (20)2 x 1000 = 3143 N
47. A spherical rain drop, falling in a constant gravitational field, grows by
absorption of moisture from the surroundings at a rate proportional to its surface
area. If it starts with zero radius, find its acceleration.
Solution
dm = K. 4

But m = 4

, where K is constant and r is the radius of the drop at any instant.

, since

dm 4
dr
dr
3r 3
4 r 3
dt 3
dt
dt

K=

dr
dt

dm
K .r 3
dt

r = Kt (since it starts with zero radius)

m
Since

and

= density of water is I gm/cc.

dv
dm
v
mg
dt
dt

4 3 dv
4
r
v.K 4 r 3 r 3 g
3
dt
3

dv
3
vK g
dt
r

But

dv
a
dt

andv=at

3
r

g
4

a+ at K =g becomes4a=g .
.. a=acceleration of drop=
48. A body moves over a plane surface, starting from rest. The frictional force is
constant. The attractive force is (P - Kt) in gravitational units. The maximum
velocity is attained after t seconds What is the distance covered before it attains
the maximum speed if the mass of body is m.
Solution

Force=ma=m

dv
dt

Kt 2
mv p Kt gdt Pt
g
2

=(p-Kt)g

d
Kt 2
Pt

dt
2
The maximum velocity is attained when

Or P-Kt= or t=(T)=

p
K

ds
Kt 2
Pt
g
dt
2

But v=

is zero.

ds
dt

Pt 2 Kt 3
g
6
2

ms
or

1 P P 2 K P3

g
m 2 K 2 6 K 3

SECTION B

g
P3
P 3
g P3
g K 3T 3 gKT 3

2 K 2 6 K 2
m
m 3K 2 m 3K 2
3m

uu
r uu
r uu
r
uur
F1 , F2 , F3 ......Fn

1. When forces

uu
r
F1

act on a particle, the particle remains in equilibrium. If

is removed then acceleration of the particle is

(a)

(c)

uu
r
F1
m

uu
r uu
r
uur uu
r
F2 F3 .... Fn F1
m

(b)

(d)

uu
r
F1

uu
r
F2
m

2. A force produces an acceleration of 4 m/sec 2 in body of mass m1 and the same


force produces acceleration of 6 m/s 2 on a mass m2. If the force is applied to a body
of mass (m1 + m2), its acceleration will be a. Then

m1 3

m2 2

m1 2

m2 3

(a)
(b)
(c) a = 2.4 m/sec2
(d) a = 5.4 m/sec2
3. n-blocks of different masses are placed on the frictionless inclined plane in
contact. They are released at the same time. The force of interaction between (n 1)th and nth block is

(a) (mn-1 - mn)g sin


(c) m

cos

(b) zero
(d) None

4. In the system shown m1 > m2. System is held at rest by thread BC. Just after
lower thread is Burnt,

(a) Acceleration of m2 is upward

m1 m2

g
m1 m2

(b) Magnitude of acceleration of both blocks will be


(c) Acceleration of m1 will be equal to zero
(d) Magnitudes of acceleration of two blocks will be non-zero and unequal
5. In the figure, R1 and R2 are reaction forces between the blocks. Which of the
following is not possible?'

(a) R2 > R1 and 5 kg block will roll


(b) R2 > R1 and 5 kg block will skid
(c) R1 >R2 and 5 kg block will roll
(d) R1 > R2 and 5 kg block will skid
6. Two blocks of 4 kg and 6 kg are attached by springs, they are hanging in vertical
position, lower spring breaks due to excessive force. Acceleration of 4 kg block just
after breaking

(a) 15m/s2
(b) 25 m/s2
(c) 10 m/s2
(d) Zero
7. A particle, initially at rest is subject to two forces; one is constant, the other is a
retarding force proportional to the particle velocity. In the subsequent motion of the
particle
(a) The acceleration will increase from zero to a constant value
(b) The acceleration will decrease from its initial value to zero
(c) The velocity will increase from zero to a maximum and then decrease
(d) The velocity will increase from zero to a constant value
8. In the adjacent figure, the spring balance and string are mass less and the pulley
is ideal. The reading of spring balance will be Balance

(a)2kg
(b) 3 kg
(c) 2.5 kg
(d) Zero
9. The block shown in the figure is in equilibrium. Find acceleration of the block just
after the string burns.

(a) 3g/5
(b) 4g/5
(c) 4g/3
(d) None
10. In the arrangement shown, the pulley is smooth and the rope is inextensible.
The reaction force between ground and the block is F. If the system is in equilibrium,
F will be (g = 10 m/s2)

(a) 50N
(b) 40 N
(c) 30N
(d) 70 N
11. A time varying force F = 2t2 is applied on a body of mass 10 kg the variation of
force with time is graphically shown.
(a) The speed of body at t = 5s is.25/3 m/s
(b) Speed of body at t = 3s is 1.8 m/s
(c) Velocity varies with time
(d) Velocity is constant
12. A time varying force applied on a body of m is F = at - bt 2 where a and b are
arbitrary The correct options is/are

(a) The force is maximum at t =

a
2b

(b) Maximum impulse is

a3
2b

a
4b

a3
12b 2

(c) Maximum force is


(d) Maximum force is
13. A mass m kg is subjected to a constant force F kg f which cause to move it in t
second to
x m. The velocity acquired is v m/s. Then distance covered x is given by

v2m
2 Fg
(a)

2 mv 2
3 Fg
(b)

(c)

2 Fgt 2
3 m

(d)

1 Fgt 2
2 m

14. The ball A, in the figure, is released from rest, when the spring is at its natural
length. For the block B of mass M to leave contact with ground at some stage, the
minimum mass of A must be:

(a)2M
(b)M
(c) M/2
(d) M/4
15. A particle of mass 2 kg is initially at rest. A force acts on it whose magnitude
varies with time as in the figure. The velocity of the particle after 10 s is

(a) 20 ms-1
(b) 75 ms-1
(c) 26 ms-1
(d) 50ms-1
16. The maximum value of the block m2 for which the system will remain in
equilibrium (coefficient of friction between block m 1 and plane surface is p), is

(a)

m1
2

(b)

m1
2

(c) m1
(d)2 m1
17. A block of mass m = (2 kg) is placed on a rough horizontal surface and is being
acted upon by a time dependent force F = 2t (where t is in second). The coefficient

of static friction between the block

and the horizontal surface is

= 0.20.The

frictional force f developed between the block and the surface versus force F plot is
as shown.
The velocity of the block at t = 4 s will be

(a) 2.5 m/s


(c) 1m/s

(b) 5 m/s
(d) 2m/s

18. In the given figure the coefficient of friction between the two blocks is and all
other surface is smooth. Find the minimum value of F which will prevent slipping.

m1 m2 m1 g

m1 m2 g

m2

m1 m2 g

m2

(a)

m1 m2 m1 g

m1

(b)

m2

(c)

(d)

19. In the given figure the coefficient of friction between the two blocks is
other surface is smooth. Find the minimum value of F which will prevent
slipping.

m1 m2 m1 g

m1 m2 m2 g

m1 m2 g

m1

m1

and all

m1 m2 g
m1

(a)
(b)
(c)
(d)
20. A particle of mass m moves on the x-axis under the influence of a force of
attraction towards the

K
i
x2

origin 0 given by F =
. If the particle starts from rest at x = a, the speed it will
attain to reach at distance x from the origin 0 will be

2K x a
m ax

1/2

2K a K
m ax

1/2

K ax
m x a

1/ 2

m a x
2 K ax

1/2

(a)
(b)
(c)
(d)
21. The block A has mass m1 and is attached to a spring having a stiffness k. The
natural length of the spring is L0. Another block B of mass m2 is pressed against

block A so the compression in the spring is d. The arrangement is released from rest
from this position. The coefficient of friction between the blocks and the ground

beneath is

. The block B will get separated from A if

2(m1 m2 ) g
k

2(m1 m2 ) g
k

(a)

d
(b)
1/2

(m1 m2 ) g
k

( m1 m2 ) g
k

(c)
(d)
22. Two blocks A and B of masses 1 kg and 2 kg respectively are placed on a
smooth horizontal surface. They are connected by a mass less inextensible string
going over a pulley as shown. The pulley is being acted upon by a vertical force of
magnitude varying with time as F =2t N. Which of the following represent the
velocity time variation of A and B?

23. Each of the two block shown in the figure has a mass m. The coefficient of
friction for all surfaces in contact is p. A horizontal force P is applied to acceleration
of block A is same in both cases is

3
4

(a) 2 mg (b) mg
(c) 3 mg (d)
mg
24. In the arrangement shown, neglect the mass of the ropes and pulley. What must
be the value of m to keep the system in equilibrium. There is no friction anywhere

(a) M

(b) 2M

(c)

M
2

(d)

M
4

25. A block of mass m is placed over a rough surface in which minimum force is

required to block, it is given coefficient of fricti

mg

1 2

mg
2

on between block and ground is

(a) mg
(b)
(c)
(d) 2 mg
26. A block of mass 0.5 kg is pulled by 12 N force on a fixed block. Speed of block is
constant. Find total contact force applied by lower block on upper block.

(a) 12N
(b) 5 N
(c )13N
(d) 17N
27. In the figure shown, the wedge is fixed and the masses are released from rest.
The coefficient of friction between 4 kg and wedge is 0.8 and between 2 kg and
wedge is 0.6. Which of the following statement is/are correct?

r
a
(a)

of blocks must be same

(b) Friction force on 4 kg is 24 N


(c) Friction force on 2 kg is 12 N
(d) Normal reaction between block is nonzero
28. Two blocks having masses m1 and m2 are connected by a thread and are placed
on an inclined plane with thread initially in a state of no tension. Thread will not
develop any tension if

(a) m1 > m2
(b) Plane is smooth
(c) No friction acts on m1 while it acts on m2
(d) No friction acts on m2 while it acts on m1
29. A block of mass 10 kg is placed in a car going down an incline of inclination 60.

1
3
If the coefficient of friction between the block and car floor is
. Find the
acceleration a of car down the incline so that the block doesn't slip on the car
surface.

g
3

2g
3

2g
2g
ora
3
3

g
2g
a
3
3

(a)
(b)
(c)
(d)
30. A lift is moving down with an acceleration 20 m/s 2. Which of the following is not
true?

(a) Acceleration of 5 kg block along incline is 15 m/s 2

3
(b) Normal force is 25
and acceleration is 5 m/s2 along incline
(c) Normal force is zero
(d) Acceleration of 5 kg is 20 m/s2 downwards

20 m/s2

31. A block is sliding along incline as shown in figure. If the acceleration to move of
chamber is a as shown in the figure. The time required to cover a distance L along
incline is:

2L
g sin acos

2L
g sin a sin

(a)

(b)

2L
g sin acos

2L
g sin

(c)
(d)
32. For a body on a horizontal surface, coefficients of static and kinetic frictions are
0.4 and respectively. When the body is in uniform motion on the surface, a
horizontal force equal in magnitude to limiting friction is applied on it. The
acceleration produced is:
(a) 0.4g
(b) 0.1g
(c) 0.2g
(d) 0.6g
33. Ten coins are placed on top of each other on a horizontal table. If the mass of
each coin is 10 and acceleration due to gravity is 10 m/s 2. What is the magnitude
and direction of the force on 7th coin (counted from the bottom) due to all the coins
above it?
(a) 0.7 N vertically downwards
(b) 0.7 N vertically upwards
(c) 0.3 N vertically downwards
(d) 0.3 N vertically upwards
34. A smooth block is released at rest on a 450 incline and slides a distance d. The
time taken to slid is n times as much to slide on rough incline than on a smooth
incline. The coefficient of friction i

s 1
(a)

k 1

1
n2

s 1
(b)

1
n2

k 1
(c)

1
n2

(d)

1
n2

35. System shown in figure is in equilibrium. Find the magnitude of net change in
the string tension between two masses just after, when one of the springs is cut.
Mass of both the blocks is same and equal to m and spring constant of both spring
is k.

(a)

mg
2

(b)

mg
4

(c)

3mg
4

(d)

3mg
2

3
36. In the figure, the wedge is pushed with an acceleration of 10
m/s2. It is seen
that the block starts climbing upon the smooth inclined face of wedge. What will be
the time taken by the block to reach the top?

2
s
5
(a)

1
s
5
(b)

5
s
2

(c)
s
(d)
37 In the figure shown A and B are free to move. All the surfaces are smooth. Mass
of A is m. Then:

(a) the acceleration of A will be more than g sin


(b) the acceleration of A will be less than g sin

(c) normal reaction on A due to B will be more than mg cos

(d) normal reaction on A due to B will be less than mg cos


38. In figure, the block moves downwards with velocity u 1, the wedge moves
rightwards with velocity u2. The correct relation between u1 and u2 is

(a) u2 = u1

(b) u2 = u1 sin

(c) 2u2sin = u1
(d) u2(1+ sin ) = u1
39. Two wooden blocks are moving on a smooth horizontal surface such that the
mass m remains stationary with respect to block of mass M as shown in the figure.
The magnitude of force P is:

(a) (M + m)g tan

(b) g tan

(c) mg cos
(d) (M + m)g cosec
40. In the system shown in figure all surfaces are smooth. Rod is moved by external
agent with acceleration 9 m/s2 vertically downwards. Force exerted on the rod by
the wedge will be

(a) 120N
(b) 200N
(c) 160N
(d) 180N
41. Two masses each m are lying on the x-axis at (a, 0) and (+a, 0) respectively
as shown figure. They are connected by a light string. A force F is applied at the
origin and along the y-axis. As a result the masses move towards each other. What
is the acceleration of each mass? Assume the instantaneous position of masses as
(x, 0) and (x, 0) respectively:

(a)

F
x
2
m a x2

(b)

F
x
2
2m a x 2

2F
m
(c)

x
a2 x2

(d)

2F
m

a2 x2
x

42. Two blocks of masses m and M placed one over the other, are being pulled by a
time-dependent force F = kt as shown in the figure. The ground is smooth but

coefficient of static friction between the blocks is


move together is

gk
(m M )
(a)

2 gk
(m M )
(b)

(c)

.The time till both the blocks

g (m M )
k

(d)

2 g (m M )
k

43. For the arrangement shown in figure the coefficient of friction between the two
blocks is p. If both the blocks are identical, then acceleration of each block is.

(a)

F
2 g
2m

(b)

F
2m

F
g
2m

(c)
(d) Zero
44. An elevator accelerates upwards at a constant rate. A uniform string of length L
and mass m supports a small block of mass M that hangs from the ceiling of the
elevator. The tension at distance 1 from the ceiling is T. The acceleration of the
elevator is

(a)

ml
M m
L

T
ml
2M m
L

(b)

ml
2M m
L

ml
2M m
L

(c)
(d)
45. A block is resting on a horizontal plate in the x-y plane and the coefficient of

friction between the block and the plate is . The plate begins to move in the xdirection and its velocity is v = bt2, t being the time and b being a constant. At what
time will the block start sliding on the plate?

b
g

gb
2

g
b

g
2b

(a)
(b)
(c)
(d)
46. Which of the following forces are electromagnetic in nature?
(a) Reaction on a body
(b) Tension in a spring
(c) Weight of a body
(d) Elastic force in a string
47. Which of the following forces are central?
(a) Gravitational force
(b) Spin-orbit interaction
(c) Force of gravity (mg)
(d) Electric force (q1,q2/r2)
48. The blocks B and C in the figure have mass m each. The strings AB and BC are
light, having tensions T1 and T2 respectively. The system is in equilibrium with a
constant horizontal force mg acting on C. Then

(a) tan

1
2

(b) tan

=1

(c) T1=
mg
(d) T2 =
mg
49. The coefficient of friction between the wheels of a car and the ground is 0.5. The
car starts from rest and moves along a perfectly horizontal road. If g = 10 ms -2, the
car
(a) can acquire a maximum acceleration of 5 ms -1 without slipping
(b) can attain a speed of 20 ms-1 in a minimum distance of 40 m
(c) can go up to a speed of 100 ms -1 in 10 sec
(d) after acquiring a speed of 50 ms-1 can come to rest, with the engine shut off and
brakes not applied in a time 10 sec
50. In case of a central force
(a) force is position dependent
(b) torque is zero
(c) angular momentum is constant
(d) force is conservative
51. If a force is conservative
(a) work is path independent
(b) it will be central
(c) work done round a closed path is zero
(d) potential energy remains constant
52. In a nucleus, the forces acting between two protons are
(a) nuclear
(b) electromagnetic
(c) gravitational
(d) weak

53. In an
-particle the forces acting between two neutrons are
(a) nuclear
(b) electromagnetic
(c) gravitational
(d)
weak
54. In an atom the forces acting between proton and electron are velocity is
(a) nuclear
(b) electromagnetic
(c) gravitational
(d)
weak
55. If a body is placed on an inclined plane, the forces acting on the body are
(a) gravitational
(b) electromagnetic
(c) nuclear
(d)
weak

56. The velocity time graph of a lift moving upwards has been shown below. Let T 1

V1 ,V2 ,V3
T2 and T3 be the tensions in the elevator cable during the three time intervals
then T1 :T2 : T3::

(a) 11: 10: 9


(b) 19: 10: 11
(c)11:10:12
(d) 11:10:8
57. A block of mass 5 kg is kept over a rough horizontal surface. A time varying
force acts on it along the horizontal given by F = 2t. The block starts slipping at t =
2.5 s and its acceleration at t = 3 s is 0.4 m/s2. The coefficients of static and kinetic
friction are respectively
(a) 0. 1, 0.08
(b) 0.08, 0.1
(c) 0.1,0.8
(d) 0. 1, 0.09
58. Which of the following is not a conservative force? (Here x and represent
position coordinates)

xi yj

yi xj

xi yj

yi xj

(a)
(b)
(c)
(d)
59. A body is acted upon by a constant force directed towards a fixed point. The
magnitude of the
force varies inversely as the square of the distance from the fixed point, then the
path can be
described by an equation similar to
(a) y = mx + C
(b) x2 +y2 = r2
(c) y = Cx2
(d) None
(where m, C and r are fixed values)

60. The minimum value of

F
mg

F
3mg

(a)

(b)

2F
3mg
(c)

between the two blocks for no slipping is:

4F
3mg
(d)

61. A block of mass m is resting on a wedge or angle


as shown a in the figure.
With what minimum acceleration a should the wedge-move so that the mass m falls
freely?

(a) g

(b) g cos

(c) g cot
(d) g tan
62. A block of mass M rests on a rough horizontal surface. The coefficient of friction

between the block and the surface is . A force F = Mg acting at an angle = 60


with the vertical side of block pulls it. In which of the following cases, the block can
be pulled along the surface.

3
2

3
2

(a)
(b)
(c)
(d)
2
3
63. Given: x = 3 + 4t + 5t + 6t and y = 7 + 8t + 9t2 + 10t3. Which of the following
is/are correct?
(a) Force is zero
(b) Force is not parallel to x-axis
(c) Force is parallel to y-axis
(d) Force is time dependent
64. In the following figures (i) and (ii), all the surfaces are smooth string is mass less
and pulley ideal. The mass of the rope is m in both the cases. If T 1 and T2 be the
tensions at the mid points the ropes in figure (i) and figure (ii) respectively, then T 1 :
T2 is

(a) m : M
(b) m : (M + m)
(c) (M + m) : m
(d) 1: 1
65. A block of mass m is placed on a prism of mass M. The inclined surface is

smooth and inclination with horizontal is . The horizontal surface is sufficiently


rough to prevent slipping of prism. The body of mass m is coming down the inclined
face then

(a) Acceleration of body along the inclined surface is g sin

(b) Frictional force is

1
2

mg sin

(c) Maximum frictional force is

Mg
2

(d) Friction force will be maximum when

= 450

66. A small body is projected with a velocity of 20.5 ms -1 on a rough horizontal

surface. The coefficient of friction ( ) between the surface changes with 0.3 time (t
in s) as the body moves along the surface. The velocity at the end of 4 s will be

(a) 5.5 ms-1


(c) 6 ms-1

(b) 5 ms-1
(d) None

67. v - t graph of an object of mass 1 kg is shown in the following figure. Which of


the following is/are correct?

(a) Net work done on the object in 30 s is zero.


(b) The average acceleration from 0 to 30 s of the object is zero.
(c) The average velocity from 0 to 30 s of the object is zero.
(d) The average force from 0 to 30 s on the object is zero.
68. A body slides over an inclined plane inclined at 450 to the horizontal. The
relationship between distance s travelled and time t is given by the equation s = ct 2,
where c = 1.73 m/sec2. The coefficient of friction between the body and the plate is
p. The acceleration down the inclined plane is a.

3
(a) p = 0.5

(b) p = 0.4

(c) a = 2

m/s2

(d) a =

m/s2

69. A man pulls a block heavier than himself with a light rope. The coef-ficient of
friction is the same between man and the ground and between block and the
ground.

(a) The block will not move unless the man moves
(b) The man can move even when the block is stationary
(c) If both move, the acceleration of man is greater than the acceleration of the
block
(d) None of the above statements is correct

m1and , m2
70. Two blocks of masses

are connected through a mass less inextensible

m1
string. Block of mass

is placed on the fixed rigid inclined surface, while the block

m2
of mass is hanging at the other end of the string, which is passing through fixed
massless frictionless pulley shown in figure. The coefficient of static friction between

m1and , m2
the block and the inclined plane is 0.8. The system of masses
from rest.

is released

(a) The tension in the string is 20 N after releasing the system.


(b) The contact force by the inclined surface on the block is along normal to the
inclined surface.

m1
(c) The magnitude of the contact force by the inclined surface on the block

is

20 3N
(d) None of these.
71. The figure shows an arrangement in which three identi-cal blocks are joined
together with an inextensible string. All the surfaces are smooth and pulleys are

a A , and , aC
massless. If

are the respective acceleration of the blocks

aB
A, B and C, then the value of

a A aC

a A , and , aC
in terms of

a A aC / 2

(a)

(b)

is

a A aC / 2

a A aC

(c)

(d)

m
72. Two blocks

and M tied together with an inextensible string are placed at rest

on a rough horizontal surface with coefficient of friction The block


is pulled with
a variable force F at a varying angle with the horizontal. The value of at which
the least value of F is required to move the blocks is given by

tan 1
(a)
insufficient data

tan 1

tan 1

(b)

(c)

(d)

m1and , m2
73. Two masses
which are connected with a light string, are placed over a
frictionless pulley. This set up is placed over a weighing machine, as shown. Three

m1 6kg , and, m2 2kg ,

m1and , m2
combination of masses

are used, in first case

m1 5kg , and, m2 3kg ,

in

m1 4kg , and, m2 4kg ,

second case
and in third case
Masses
are held stationary initially and then released. If the readings of the weighing

w1 , w2 and w3
machine after the release in three cases are

W1 W2 > W3
(a)

W1 W2 W3
(b)

W1 W2 W
(c)

respectively then

W1 W2 W3
(d)

a0 2m / s 2
74. The pulley is given an acceleration

starting from rest. A cable if

connected to a block A of mass 50 kg as shown. Neglect the mass of the pulley. If


=. 0.3 between the block and the floor, then the tension in the cable is

(a) 200 N
(c) 300N

(b) 250N
(d) 350N

75. A solid cube of mass 5 kg is placed on a rough horizontal surface, in xy-plane as


shown. The friction coefficient between the surface and the cube is 0.4 An external

ur

F 6i 8 j 20kN

force
is applied on the cube. (use g = 10 m/s 2). Positive z-direction
is taken vertically upwards as shown. Then choose the correct statement (s).

(a) The block starts slipping over the surface


(b) The friction force on the cube by the surface is 10 N.

3i 4 j
(c) The friction force on the cube acts along the vector

10 10N
(d) The contact force exerted by the surface on the cube is

76. Two men of unequal masses hold on to the two sections of a light rope passing
over a smooth light pulley. Which of the following are possible?

(a) The lighter man is stationary while the heavier man slides with some
acceleration.
(b) The heavier man is stationary while the lighter man slides with some
acceleration.
(c) The two-men slide with the same acceleration in the same direction.
(d) The two men slide with the accelerations of the same magnitude in opposite
directions.

F1 and F2
77. Two forces

F1
act on a thin uniform elastic rod placed in space. Force

F2
acts at right end of rod and
acts exatly at centre of rod as shown (both forces act
parallel to length of the rod).

F1
(a)

F2
causes extension of rod while

F1
(b)

causes compression of rod.

F2
causes extension of rod and

F1

also causes extension of rod.

F2

(c)
causes extension of rod while
F2 does not change length of rod.
The correct order of True / False in above statements is
(a) T F F
(b) F T F
(c) F F T
(d) F F F
78. A block of mass 5 kg is placed over a rough horizontal surface with coefficient of

friction
= 0.1. There is a horizontal force of 13 N is acting on it towards right and
another horizontal force F is acting on it towards left. Taking g = 10 m/s 2, the range
of over which block does not slip is

8 F 18 N

0 F 18 N

(a)
(b)
(c) any value less than 18 N
(d) Any value above 18 N
a0
79. Two blocks A and B each of mass m are placed on a smooth horizontal surface.

Fand 2F
Two horizontal force
are applied on the two blocks A and B respectively as
shown in figure. The block A
does not slide on block B. Then the normal reaction acting between the two blocks
is

(a) F

(b)

F
3

F
2

(c)

(d) 3F

80. A boy of mass 40 kg wants to climb up a rope hanging vertically. The rope can
withstand a maximum tension of 500 N. The maximum acceleration with which a
boy can climb the rope is
(a) 1.5 m/s2 (b) 2 m/s2
(c) 2.5 m/s2 (d) 3 m/s2
81. A particle is placed at rest inside a hollow hemisphere of radius R. The
coefficient of friction

between the particle and the hemisphere is


which the particle
can remain stationary is:

(a)

R
2

1
(b)

3
R
2

1
3
The maximum height upto

3
R
2
(c)

(d)

3R
8

l1
82. The length of an elastic string is

l2 F2

F1
when stretched by a force

and the length is

F2
when the stretching force is

. The length of the string, when it is stretched

F1 F2 ,
by force

is

F2l2 _ F1l1
F2 F1

l1 l2
(a)

(b)

F2l2 _ F1l1
F2 F1

F2l1 _ F1l2
F2 F1

(c)

(d)

83. If the coefficient of friction between an insect and bowl is


and the radius of
the bowl is r, the maximum height to which the insect can crawl in the bowl is

1
r 1

r 1 2

r
1 2
(a)

(b)

r 1 2

1 2 1

(c)
(d)
84. Two bodies of mass m and 4m are attached with string as in the figure. The body

l
of mass m hanging from a string of length is executing oscillations of angular

0
amplitude
while the other body is at rest.
What is the minimum coefficient of friction between the mass 4m and the horizontal
surface?

(a)

(c)

2 cos 0

1 cos 0

2 cos 0
(b)

(d)

3 2 cos 0

85. A bar moves with an acceleration a parallel to the ground. A ring can slide on

the bar. If coefficient of friction between the bar and ring is ,


the bar due to the ring can be

the frictional force on

(a)

ma

(b) ma cos

m g 2 a2

(c)

mg

(d)

86. Which of the following is true?


(a) Rolling friction appears due to deformation at the point of contact
(b) All perfectly rigid wheels experience rolling friction
(c) For identical surface in contact coefficient of static friction > coefficient of sliding
friction
(d) Friction is component of total contact force parallel to surface of contact

87. A chain of mass per unit lengh

and lenght 1.5 in rests on a fixed smooth

sphere of radius
in such that A end of chain is at the top of sphere while the
other end is hanging freely. Chain is held stationary by a horizontal thread PA. The
tension in the thread is

1 2

2

(a)

2

2

(b)

(c)

2
g

(d) None

88. Three blocks A, B and C of masses 2 kg, 3 kg and 4 kg are placed as shown.
Coefficient of friction between A and B is 0.5 and that between B and C is 0.1. The
surface is frictionless. The maximum force F that can be applied horizontally onto A
such that the three blocks move together is

(a) 12.22 N
(c) 11.25 N

(b) 13N
(d) None

m1 and m2
89. Two masses
a

are connected by a massless inextensible string going over

m1 , m2
pulley. If the accelerations of ,
and pulley w .r .t. ground be
which of the following is correct (assume only gravity forces)?

(a)
(c)

r r
r
a1 a 2 2a p

r r
r
a1 a 2 o

(b)

r r
r
a1 a 2 a p

r r
r
a1 a p a 2
(d)

r r
r
a1 , a 2 and a p
then

90. A block of mass 2 kg is to be lifted with constant velocity by applying force F

down the rope that passes over a pulley having coefficient of friction
required is

2ge

(a)

2ge
(b) 2g

(c)

2ge

The pull

(d)

91. In the system shown, the mass m = 2 kg oscillates in a circular arc of amplitude
60, the minimum value of coefficient of friction between mass = 8 kg and surface
of table to avoid slipping is

(a) 0.25

(b) 0.50

(c) 0.40

(d) None

92. A long pliable carpet is laid on ground. One end of the carpet is bent back and
pulled backwards with constant velocity 16 m/s. If mass/length of carpet is 1 kg/m,
the minimum force needed to pull the moving part is

(a) 4 N
(b) 8 N
(c) 16 N
(d) 128 N
93. A flexible chain of mass m hangs between two fixed points A and B at the same
level. The inclination of the chain with the horizontal at the two points of support is
The tension at the mid point C of the chain is

(a)

mg
2 tan

(b)

mg
2 tan

mg
(c) Zero

(d)

(sin cos)
2

SECTION C
C-I
Two blocks A and B of masses 10 kg and 12 kg respectively are kept on a rough
wedge of inclination 30 and 600 respectively. The coefficient of friction between the
block A and wedge is 0.6 while that between B and the wedge is 0.3. The blocks
areA connected by a light inextensible thread. The wedge is fixed with respect to
ground.

1. The acceleration of block A is

(a)

(c)

6 35
g
22

up the plane

53 3
g
10

(b)

6.8 3 3
g
22

down the plane

down the plane (d) Zero

2. Tension in the thread connecting A and B is (Take g = 10 m/s 2)


(a) 24.83 N
(b) 85.92 N
(c) Zero
(d) 55.79 N
3. Force of friction acting on the block A is (Take g = 10 m/s 2)
(a) 35.92 N (b) 25.79 N (c) 51.96 N (d) 54.83 N
C-2
A block of mass m sits on the top of an identical block which sits on the top of a flat

1
rough The coefficient of static friction between the surfaces of the blocks is

2
coefficient of friction between the block and table is

and

1. A horizontal force F is applied to the top block only, and the force is increased
until the top starts to slide. The bottom block will slide with top block only if

1
1 2
2

1
2 1 2
2

1
2 1 2
2

2 2 1

2 2 1

2 1

(a)
(b)
(c)
(d)
2. Now instead, a horizontal force F is applied to the bottom block only and this
force is in until the bottom block just starts to slide. For the top block to slide with
the bottom block.

(a)

(b)

1 0
(c)

2 1
(d)

3. If m = 5 kg;
= 0.6 and
= 0.2. Then the minimum force to be applied on the
top block to produce motion in the system of blocks is (take g = 10 m/s 2)
(a) 20N
(b) 17.14N (c) 19.61N (d) 51.44N
C-3
Consider a system of three equal masses and 4 pulleys arranged as shown. The
mass of each block is m

1. The acceleration of C is

(a)

2g
23

downwards

(b)

2g
23

g
4

(c)
upwards
2. Acceleration of B is

(a)

(c)

2g
9
7g
9

(b)

(d)

upwards

(d)

g
4
g
9

g
9

upwards

3. Tension in the single thread runing through all the pulleys is

(a)

5
9
3
4

mg

(b)

25
23

mg

7
9

(c)
mg
(d) mg
C-4
A body with a mass m slides along the surface of a trihedral prism of mass M, whose
upper plane
h table.
is inclined at an angle to the horizontal. The prism rests on a
horizontal plane having a vertical of static wall at the rear edge of the prism to keep
it at rest.

1. The force exerted by the base of the prism on the horizontal plane is Ltsncreased
(a) Mg
(b) mg
(c) Mg + mg sin2 (d) Mg + mg cos2
2. The force exerted by the wall prism
(a) Mg
(b) mg
(c) mg Cos (d) mg sin cos
3. The normal reaction on the block is
(a) mg
(b) Mg
(c) mg cos (d) mg sin cos
C-5
The three flat blocks as shown in the figure are placed on the 30 incline and a force
P = 80 N parallel to the inclined plane is applied to the middle block. The upper
block is prevented from moving by a wire which is attached to the fixed support.
The coefficient of static friction for each of the three pairs of surfaces is shown in
the figure.

1. Normal reaction between the 40 kg block and the inclined plane is (g= 10m/s 2)

(a) 150,f3 N (b) 250,f3 N (c) 400-N[3 N


. (d) 6006N
2. Normal reaction between the 50 kg block and the 40 kg block is

150 3N

250 3N

400 3N

600 3N

(a)
(b)
(c)
(d)
3. If the force applied is 80 N, then study the following statements:
(i) slipping occurs between 40 kg block and the inclined plane
(ii) the 50 kg block slips and 40 kg block remains in place
(iii) slipping occurs between 40 kg block and the 50 kg block
(iv) the 40 kg and 50 kg blocks move together
Correct statements are: (a) (i) and (ii) only (b) (i) and (iii) only (c) (i) and (iv) only (d) (ii) and (iv) only
C-6
The system consists of a msss A of 40 kg, mass B of 15 kg and a force F is applied
parallel to the smooth inclined plane.
Friction exists between A and B Take g = 10 ms-1.

1. A minimum force F = F1 is applied on block A so that the block A justs stays in


equilibrium. At this instant the frictional force between A and B is
(a) 75 N
(b) 50 N
(c) Zero
(d) 200 N

F 2 F1
2. When the force applied on block A is

it is found that the block B just

begins to slide A. What is the value of coefficient of friction

(a)

3
5

(b)

1
3 1

1
3
(c)

3
(d)

F 2 F1
3. When

what is the normal reaction between A and B?

( 3 1)
(a) Zero
(b) 75, N
(c) 75
N
(d) 187.5 N
4. If = 0, then acceleration of B relative to ground is
(a) 5 ms-2
(b) 2.5 ms-2 (c) 1 ms-2
(d) Zero
C-7

Static friction act when there is no relative motion, which is self adjusting from zero

S
to maximum value

N. It depends on normal force N. While kinetic friction force is

k
constant and given by

one

N.Sometimes

k
and

are not given separately but only

is given In that case maximum static friction is equal to kinetic friction

N.

Block B rests on a smooth surface and coefficient of friction between A and B is


0.4 as in figure. (Take g = 10 m/sec2)

1. Acceleration of a block A if F = 30 N
(a) 1.213 m/s2
(b) 2. m/s2
2
(c) 0.857 m/s
(d) 1.642 m/s2
2. Acceleration of block B if F = 30 N
(a) 1.213 m/s2
(b) 1.642 m/s2
2
(c) 2.124 m/s
(d) 0.857 m/s2
3. Acceleration of block A if F = 250 N
(a) 21 m/s2
(b) 1.6 m/s2
(c) 0.857 m/s2
(d) None
4. Acceleration of block B if F = 250 N
(a) 10 m/s2
(b) 1.6 m/s2
(c) 8 m/s2
(d) None
C-8
to slide on As shown in figure, a wedge of mass m 3 = 3.45 kg is placed on a smooth
horizontal surface. A small and light pulley is connected on its top edge. A light
flexible thread passes over the pulley. Two blocks having mass m 1 = 1.3 kg and m =
1.5 kg are connected at the ends of the thread m 1 is on smooth horizontal surface
and m2 rests on inclined surface of the wedge. The whole system is released from
rest. (g = 10 m/s2)

1. The tension in the string is


(a) 4.6N
(b) 3.9N
(c) 6.2N
(d) 8.4N
2. Acceleration of m1 is
(a) 3 m/s2
(b) 2 m/s2
2
(c) 4 m/s
(d) 5 m/s2
3. Acceleration of m3 is
2 m/s2
(a) 3 m/s2
(b) 2 m/s2
2
(c)4 m/s
(d) 5m/s2
C-9
Two blocks of mass m = 5 kg and M = 10 kg are connected by a string passing over
a pulley B as shown. Another string connects the centre of pulley B to the floor and
passes over another pulley A as shown. An upward force F is applied at the centre of
pulley A. Both the pulley are massless. Find the acceleration of blocks m and M (in
m/s2).

1. If F is 100
(a) 5,0
(b) 5,5
(c) 0,0
(d) 5,2.5
2. If F is 300N
(a)5,0
(b)5,5

(c) 15, 7.5 (d) 15,2.5


3. If F is 500N
(a) 5, 0
(b) 2.5, 5
(c) 5,7.5
(d) 15,2.5
C-10
As shown in figure pulleys are small, light and frictionless, threads are inextensible
and mass of blocks A, B and C is m 1, = 5 kg, m2 = 4 kg and m3 = 2.5 kg respectively.

Coefficient of friction for both the planes is

= 0.50.

1. Acceleration of block A is
(a) 3 m/s2
(b) 2 m/s2
c) 4 mis2
(d) None
2. Acceleration of block B is
(a) 3 m/s2
(b) 2 m/s2
(c) 4 mIs2
(d) None
3. Acceleration of block C is
(a) 3 m/s2
(b) 2 m/s2
(c) 4 m/s2
(d) None
C-11
Three masses M, M and 2M are held stationary as shown in the figure with all the
strings taut. Now, the mass 2M is released and it starts falling vertically downward.

1. Velocity of masses Mat the instant 2M touches the ground is

10
(a)

5
13

15
m/s

(b)

5
17
m/s

10

15
7

10

5
17

(c)
m/s
(d)
m/s
2. Velocity of mass 2M at the instant it touches the ground is

6
(a)

5
13

6
m/s

10

(b)

5
17
m/s

15
7

(c)
m/s
(d) 10 m/s
3. The maximum height reached by masses M (in metre is)
(a) 0.78
(b) 1.53
(c) 2.16
(d) None
C-12
A small sphere is suspended by a light string from the ceiling of a car and the car
begins to move with a constant acceleration a.
1. The tension produced in the string is
(a)T =T0
(b) T > To
(c)T < T0
(d)T = 0
where T0 is the tension in the string when the car is at rest or moving with uniform
velocity.
2. The value of the tension produced in the string is

m g 2 a2

m g 2 a2

m g 2 2a 2

(a)mg (b)
(c)
(d)
3. The inclination of the string to the vertical is
(a) tan-1 (a/g) in the direction of motion
(b) tan-1 (a/g) opposite to the direction of motion
(c) tan-1 (g/a) in the direction of motion
(d) tan-1 (g/a) opposite to the direction of motion
C-13
A plumb line is suspended from the roof of a railroad car and the car is moving on a
circular road.
1.The plumb line inclines
(a) forward
(b) rearward
c) towards the centre of the path
(d) away from the centre of the path
2. The inclination of the string of the plumb line w.r.t. vertical direction is

tan 1 (rg/ v 2 )

tan 1 (v 2 rg)
(a)

(b)

tan (v / rg)
1

tan 1 (v 2 r/ g)

c)
(d)
3 The tension in the string of the plumb line is

v 2
m g
r

v2
m g
r
(a)

(b)

v
m g
r

(c)

v 2
m g
r

(d)

C-14
Two masses 40 kg and 30 kg are connected by a massless string passing over a
frictionless pulley as shown in the figure.

1. The tension in the string will be


(a) 188N
(b) 368N
(c) 288N
(d) 168N
2. The acceleration of the system will be
(a) 0.5 ms-2 (b) 0.6 ms-2 (c) 0.7 ms-2 (d) 0.8 MS-2
3. If m1 = m2, acceleration of the system will be
(a) zero
(b) 0.2 ms-2 (c) 0.4 ms-2 (d) 0.6 ms-2
C-15
Block A of mass 35 kg is resting on a frictionless surface and another block B of
mass 7 kg is resting on it as shown in the figure. The coefficient of friction between
the blocks is 0.5 while kinetic friction is 0.4. A force of 100 N is applied to block B. (g
= 10 MS-2) AIt- 35 kg

1. The acceleration of the block A will be


(a) 0.8 MS-2 (b) 2.4 ms-2 (c) 0.4 MS-2 (d) 4.4 MS-2
2. The minimum value of force to cause block B to slip over block A is
(a) 72 N
(b) 42 N
(c) 35 N
(d) 60 N
3. If m = 10 kg, m9 = 40 kg and applied force is 40 N, the acceleration of the block
B with respect to block A will be

(a) zero
(b) 0.5 MS-2 (c) 2.5 ms-2 (d) 0.8 MS-2
4. If mA = 10 kg, mB = 40 kg and applied force is 40 N, what will be the acceleration
of the system
(a) zero
(b) 0.5 MS-2 (c) 5.0 MS-2 (d) 0.8 MS-2
C -16
The coefficient of friction between m2 and inclined plane is p as shown in the figure.

m1
m2

1. If
=sin ,
(a) no motion takes place
(b) m1 moves downward
(c) m1 moves upward
(d) data insufficient
2. m1 starts coming down if

m1
sin cos
m2
(a)

m1
sin cos
m2
(b)

m1
sin cos
m2

m1
sin cos
m2

(c)
(d)
3. What is the acceleration of the pulley P4?

m1
sin cos
m2
(a)

m1
sin cos
m2
(b)

m1
sin cos
m2
(c)

(d)data insufficient

C-17
At t=0, a variable force varying with rescept to time as F=6t-2t 2 starts acting o0n a
2 kg body which is initially at rest (F is in Newton and t is in sec) When the boby
comes to rest again (we can see that at t=0, F =0) tehe force is withdrawn.

1.Find the time interval for which the force acts on the body
(a)2s (b)3s (c)3.5s
(d)4.5s
2.What is the time when the velocity of the body is maximum?
(a)2s (b)3s (c)3.5s
(d) 4.5s
3.Mark the correct statement:
(a)he velocity of the body is maximum when the force acting on the body is
maximum for the first time
(b)The velocity of the body becomes maximum when the force acting on the body
becomes zero again
(c)When the force becomes zero again, the velocity of the body also becomes zero
at the instant.
(d)All of the above
C-18
All the pulleys and strings are light and also all the surfaces are smooth. Take and
also all the surfaces are amooth .Take 10 ms-2

1. Find the tension in string attached to the block of mass m


(a)40N
(b)10N
(c)20N
(d)5N
2.What is the acceleration of the block M?
(a)4.5ms-2 down the incline
(b)4.5 ms-2 up the incline
-2
(c)5 ms down the incline (d)5ms-2 up the incline
3.What is the accerlation of the pulley p4?
(a)2.25ms-2 towards left (b)2.25ms-2towards right
(c)9ms-2 towards left
(d)9ms-2towards right
C-19
A block of mass m is pushed by a force F, as shown in figure making angle 0 with

vertical. The coefficient of friction between block and surface is

1. The minimum force required to slide the block is

mg
sin cos

mg
sin cos
(a)

(b)

mg
sin cos

mg
cos sin
(c)

(d)

2. If angle of friction is

mg sin
sin( )

, then the force required to pull is

mg sin
sin( )

(a)

(b)

mg sin
sin( )

mg cos
sin( )

(c)
(d)
3. For what relation between angle 8 and angle of friction a, the block cannot move
whatever
may be the force applied
(a)
(c)

<

(b)

>

(d) None

C-20
In the adjacent figure, x-axis has been taken down the inclined plane and y-axis

perpendicular to the inclined plane. The coefficient of friction varies with


where k = tan . A block is released at 0.

= kx,

1. The maximum velocity of block will be

g sin

g
(a)

(b)

gcos

g tan

(c)
(d)
2. Maximum distance travelled by the block is

1
2

(a) 1 m
(b) 2 m
(c) 3 m
(d)
m
3. Frictional force acting on the block after it comes to rest

mg sin
2

mg sin
2

(a) mg sin (b) 2mg sin


(c)
(d) 2mg cos
4. Frictional force acting on the block just before it comes to rest

(a) mg sin (b) 2mg sin


(c)
(d) 2mg cos
C-21
The following figure shows a system of two blocks constrained to move in horizontal
by means of light strings and smooth, frictionless pulleys. A force F pulls on to
bigger block. The masses are same and there is no friction anywhere. Let a1 and a,
be the accelerations of two blocks.

1. The correct relation between

and

(a) 2 1 = 2
(b) 2 1 = - 2
(c)
2. The acceleration of smaller block is

(a)

2F
, left
5m

(b)

2F
, right
5m

(c)

F
, left
5m

is

(d)

1=2

F
, right
5m

(d)

1=-2

3. The tension in the string is

(a)

2F
5

(b)

F
5

(c)

3F
5

(d)

4F
, right
5

C-22
A block is kept over a rough horizontal ground with coefficient of friction p. At the
instant shown, block is at rest. Take this moment as t = 0. At this moment, a time
dependent force given by F = F0 e-t/r starts acting
m on the block along horizontal.
Here, rand F0 are known constants. The mass of the block is m.

1. The minimum value of F0 so that the block starts slipping over the surface is

mg

mg
(a)

1 2

(b)

1 2 mg

mg

(c)
(d)
2. If the acceleration of the block becomes zero at t = i, then initial acceleration of
the block is

(a)

(c)

F0
m

F0
em

(b)

(d)

F0
1
1
m
e

F0
1
1
em
e

3. The velocity time graph of the block will be like

C-23

A particle slides down a smooth inclined plane of elevation fixed in an elevator

0
going up with an acceleration

. The base of the incline has a length L.

1. The acceleration of particle with respcet to the incline


(a)g sin

(b)

sin

(c) (g +
)sin
(d) (g sin + a0cos )
2. The time taken by the particle to reach the bottom

2L
t

g sin

(a)

2L

0 sin

1/2

(b)

2L

( g 0 )sin cos

1/2

2L

( g 0 )sin

1/2

1/2

(c)
(d)
3. If the elevator going up with constant velocity, the time taken by the particle to
reach the bottom is

(a)

2L

( g sin cos

1/2

(b)

2L

( g sin

1/2

(c)

2L

( g cos

1/2

(d)None

C-24
If two masses A and B drawn in their attached cables with an acceleration of

0.2
t m / s 2
3

where t is in seconds

1. The speed of the block when it reaches a height of h = 4 m starting


from rest is
(a) 4 m/s
(b) 2 m/s
(c) 1 m/s
(d) None
2. The speed of pulley D is
(a) 2.5 m/s(b) 3.5 rn/s(c) 4.5 rn/s (d) None
3. If vA=kvD Then k is
(a) 2 (b) 3 (c) 4 (d) None
C-25
A car starts from rest with a constant acceleration of 2 m/s 2. After 5 s, a ball is
dropped through the window of the car. The window of the car is at a height of 1.25
m from the ground.
1. What will be the speed of the ball, 0.5 s after it was dropped? (Take g = 10 m/s 2)

5 5

(a) 5 m/s
(b)
m/s
(c)
m/s (d) 2 m/s
2. The acceleration of the ball after it is dropped is given by

(10) 2 (2) 2
(a) 10 m/s2 (b)
m/s2
(c) 12 m/s2 (d) 2 m/s2
3. The angle which the velocity vector makes with the vertical is

(a) tan-1

(c) tan

1

2

-1

(b) tan-1(2)

1

5

(d) tan-1(5)

C-26
A helicopter of mass M is rising vertically upwards with a uniform acceleration a. If
the mass of the pilot in the helicopter is m.
1. What is the magnitude and direction of the force exerted by the pilot on the floor
of the helicopter?

(a) m (g +

) vertically downwards

a
(b) m (g -

) vertically upwards

(c) (M + m) (g+ ) vertically upwards

(d) (M + m) (g+ ) vertically downwards


2. What is the magnitude and direction of the action force exerted by the helicopter
on the surrounding air?

(a) m(g +

) vertically downwards

(b)m(g -

) vertically upwards

(c) (M + m) (g +

) vertically upwards

(d) (M + m) (g + ) vertically downwards


3. What is the magnitude and direction of the reaction force exerted by the
surrounding air on the
helicopter with pilot inside?

(a) m(g +
(b) m(g -

)vertically downwards
) vertically upwards

(c) (M + m) (g +
(d) (M + m) (g +
3.

) vertically upwards
) vertically downwards

If VA = kvD Then k is
(a) 2 (b) 3 (c) 4 (d) None

C.25

1.
m/s2)

A car starts from rest with a constant acceleration of 2 mIs 2. After 5 s, a ball is
dropped through the window of the car. The window of the car is at a height
of 1.25 m from the ground.
What will be the speed of the ball, 0.5 s after it was dropped? (Take g = 10

5
2.

5 5

(a) 5 m/s
(b)
m/s (c)
m/s (d) 2 m/s
The acceleration of the ball after it is dropped is given by

10
3.

(a) 10 m/s2 (b)


m/s2
(c) 12 m/s2 (d) 2 m/s2
The angle which the velocity vector makes with the vertical is

(a)

(c)

1
tan 1
2
1
tan 1
5

tan 1 (2)
(b)

tan 1 (5)
(d)

C-26

1.

2.

3.

A helicopter of mass M is rising vertically upwards with a uniform acceleration


a. If the mass of the pilot in the helicopter is m.
What is the magnitude and direction of the force exerted by the pilot on the
floor of the helicopter?
(a) m (g +a) vertically downwards
(b) m (g - a) vertically upwards
(c) (M + m) (g +a) vertically upwards
(d) (M + m) (g + a) vertically downwards
What is the magnitude and direction of the action force exerted by the
helicopter on the surrounding air?
(a) m(g +a) vertically downwards
(b) m(g - a) vertically upwards
(c) (M + m) (g + a) vertically upwards
(d) (M + m) (g + a) vertically downwards
What is the magnitude and direction of the reaction force exerted by the
surrounding air on the helicopter with pilot inside?
(a) m(g +a).vertically downwards
(b) m(g - a) vertically upwards
(c) (M + m) (g + a) vertically upwards
(d) (M + m) (g + a) vertically downwards
SECTION D

Column.I
(a) Central forces

Column.II
(p)Electromagnetic in nature

2.

dU
dx

(b) Frictional forces


(q)
(c) Conservative forces
(r) Non-conservative
(d) Gravitational force
(s) Elliptical orbits
Column-I
Column-II
(a) Block of mass 2kg on a rough
(p) Tension at the mid point of block is

10 N
horizontal surface pulled by a horizontal

20 N , SL 0.5
force of

(b) Block of mass 2 kg pulled with


(q) Acceleration of block is 5 mIs2
constant speed up an incline of inclination

1
3
300 and coefficient of friction
(c) Block of mass 0.75 kg pulled by a
constant force of 7.5 N upon incline of

(r) Force of friction acting is 5N

1
3
inclination 300 and coefficient of friction
(d) Block of mass 2 kg pulled vertically by

(s) Resultant force on the block


is zero
3.
Two forces F1 and F2 are acting on system according to block.

fr1
Friction force between blocks

fr2
T

4.

Friction force between lower block and ground


Tension in string

Column-I
Column-II
(a) If F1= 8N,F2 = 18N
(p) fr1 = 8
(b) If F1 = 12 N, F2 = 12 N
(q) fr1 = 4
(c) IfF1=4N,F2=20N
(r)fr2=10
(d) IfF1=8N,F2 =0N
(s)T=0
A block of 10 kg is placed on rough inclined surface of coefficient of friction
= 0.8.

Column-I
(a) F= 30N,

Column-II

=30 (p) Body is at rest

(b) F = 30 N,
(c) F = 30 N,

= 00

(q) Body is in acceleration

= 90

(r) N= 86N

5.

6.

(d) F = 50 N,
= 30
(s) N= 101 N
Two similar blocks are connected by a string. Forces of 4 N and EN are applied
on blocks according to diagram. T represents tension in string. F 1 represents
friction on left block and f2 represents friction on right block.

Column-I
Column-II
(a) F=0N
(p) f1 = 4
(b) F=4N
(q) f2=0
(c) F=20N (r) T=0
(d) F=1ON (s) f2 = 4
A pulley system is attached to an elevator as shown in figure. The elevator
starts to move up with an acceleration a.

Column-I

Column-II

1
(m m )
2 2 1 1 at 2
(m1 m2 ) 2
(a)Acceleration of m1 in elevator frame

(b)Acceleration of m in ground frame

(p)

(q)

(a)Distance covered of m1 in elevator frame (r)

1 m 2 m1
( g a) t 2

2 (m1 m2 )

m 2 m1

ga
m1 m2

(m 2 m1 )
( g a)
(m1 m2 )
(d)Distance covered of m1 in ground frame
7.

8.

(s)

Column-I
Column-II
(a)Strong
(p)Graviton
(b)Electromagnetic
(q)Photon
(c)Weak
(r)Boson
(d)Gravitational
(s) Pion
In the figure, all pulleys are massless and strings are light.

Column-I
(a)1 kg block
(b)2 kg block
(c)3 kg block
(d)4 kg block
9.

Column-II
(p)will remain stationary
(q)will move down
(r)will move up
(s)5m/s2
(t)10rn/s2
In the figure (g = 10 mIs2)

Column-I
(a)Acceleration of 2 kg block
(b)Net force on 3 kg block
(c)Normal reaction between
2 kg and 1 kg
(d)Normal reaction between
3 kg and 2 kg

Column-Il
(j)8 SI unit
(u)25 SI unit
(r)2 SI unit
(s)45 SI unit

(t)None
1.
Statement-I : Normal contact force offered by a horizontal surface on a block
placed on it
doesn't form an action-reaction pair with the weight of the block.

Statement-2: Contact force is electromagnetic in nature and weight is


gravitational.
2.
Statement-I: When a block moves on a rough horizontal surface with some
speed, it eventually slows down.
Statement-2: Friction always opposes motion.
3.
Consider a block of mass m kept on a rough horizontal surface..
Statement-I: It is possible to apply a non-vertical force of however large
magnitude without disturbing the equilibrium of block.
Statement-2: If no force is applied, force of friciton is zero.
4.
A system consisting of two blocks kept one over the other rests over a
smooth horizontal surface. Somehow it is set in motion so that the system of
blocks acquires a constant velocity. Friction coefficients between the two

5.

6.

9.

11.

blocks is
( # 0)
Statement-1 : Afterwards, friction between the blocks is static in nature and
non zero. Statement-2: The lower block is in translational equilibrium.
Statement-I: While walking on a slippery road, one should take shorter steps.
Statement-2: For longer steps, the line joining the centre of gravity of man to
the toe of his foot makes a larger angle to the vertical. This decreases the
normal reaction and thus friction.
Statement-I: Kinetic Friction force opposes the relative motion of the body.
Statement-2: Friction force is generated due to relative slipping between
bodies. Statement-i: For motion of body, we have to apply force.
Statement-2: To change position of a body, velocity is necessary.
Statement-1 : Reading of a weighing machine is not always equal to weight of
the body. Statement-2: Reading of weighing machine is equal to normal
reaction on body by weighing machine.
Statement-1 : Normal reaction acting on a body is always perpendicular to
the sufrace. Statement-2: Normal reaction is a component of contact force.
Statement-I: Static friction is always equal to applied force.
Statement-2: For static friction, body must be at rest.
Statement-1 : In the figure shown, upper block is always relatively at rest
w.r.t. lower block, whatever be the magnitude of force.

Statement-2: Maximum acceleration of upper block is pg, where p =


coefficient of friciton between upper and lower block.
12.
In the diagram, spring is connected between two blocks and it is in
compressed state. In
Statement-1 : Normal reaction on lower block by ground is 2mg.
Statement-2: In equilibrium, net external force must be zero.
-,

13.

14.

15.

Statement-1 : According to the Newton's third law of motion, the magnitude


of the action tion force in an action reaction pair is same only in an inertial
frame of ref Statement-2: Netwon's laws of motion are applicable in every
inertial reference frank
Consider a system of two blocks each of mass m, placed over a smooth
surface. A is applied to move the blocks together either on A or B.
Statement-I : The normal contact force between the two blocks will be of
same ma4 two situations.
Statement-2: The normal contact force between the blocks does not depend
on the f4. force has been applied on A or B."
Statement-1 : A particle always moves in the direction of net force acting on
the p vided it starts from rest.
Statement-2: When a body starts from rest with uniform acceleration, its

r r
r r
v at.So, v || a

velocity
16.
Statement-I: A body can possibly move with constant speed under the action
of
Statement-2: A single force can be treated as resultant of two oppositely
directed forces
17.
A block of mass m is suspended from a light, inextensible string from the
ceiling of*
Statement-I: The tension in the string is less than the weight of the block,
when goes down.
Statement-2: When the block experiences a downward acceleration, net force
directed downward.
18.
Statement-1 : In a frame of reference attached to a particle itself, the
acceleration is zero.
Statement-2: The frame of reference which moves with constant velocity
w.r.t. reference is an inertial frame of reference.
19.
Statement-1 : A body can be pulled with the least effort if it is pulled at an
angle of friction from the surface.

Statement-2: If coefficient of static friction is

, the angle of friction is

tan 1 ( )
20.

21.
22.

Statement-1 : Instantaneous acceleration of a particle on a curved path may


be along the instantaneous velocity vector.
Statement-2: Instantaneous velocity is always tangential to the path.
Statement-I: Newton's second law of motion is the main law of motion.
Statement-2: Newton's first and second law are contained in second law.
Statement-I: A body under the action of two forces is in equilibrium.
Statement-2: The forces must be collinear.

23.

24.

25.

Statement-.I : Force of action and reaction do not cancel each other, each
produces its own effect.
Statement-2: Forces of action and reaction are equal and opposite.
Statement-I: The apparent weight of a body in an elevator moving with some
downward acceleration is less than the actual weight of body.
Statement-2: The part of the weight is spent in producing downward
acceleration, when body is in elevator.
Statement-I: When we press the ground harder we can walk rapidly.
Statement-2: Frictional force will be more when we push the ground harder,
which pushes us with more acceleration.

SECTION F
1. The block of mass 1 kg placed on a smooth wedge of inclination 37 which
gets a horizontal acceleration a to the right. Find acceleration of block 1 kg
w.r.t. wedge m when

a
2.

3a
5

g
4

3g
4

(a)
(b)
(c)
A block of mass 2 k slides down an inclined plane which makes an angle of

3
2

30 with The coefficient of friction between the block and the surface is (a) What force must be applied to the block so that the block moves down
the p acceleration?
(b) What force should be applied to the block so that it can move up without
any ac
(c) Calculate the ratio of powers needed in the above two cases, if the block
moves 'speed in both the cases.
3.
A plank of mass 4 kg is placed on a smooth horizontal surface. A block of
mass 2 k the plank and is being acted upon a horizontal force F = 0.5 t where
F is in newton If the coefficient of friction between the block and the plank is
0.10 then find the velocity of 2 kg block at t = 10 s.

4.

The two blocks shown in figure are rough and coefficient of friction between

them is
= 0.4. The blocks are given velocities of 2m/s and 8 m/s in the
directions as in figure (g = 10 mIs2). Find:

5.

(a) The time when relative motion between them will stop.
(b) The common velocities of blocks upto that instant.
(c) Displacements of 1 kg and 2 kg blocks upto that instant.
A block of mass M is pulled by a uniform string of length I and

linear mass density


by the application of a horizontal force F. Find the
variation of tension in the string with x.

6.

7.

A vertical force of magnitude F acts at the top of a string of mass m and


length I . if a body of mass M hangs at the bottom of the string, find the:

(a) acceleration of the system (M + m),


(b) variation of tension in the string with x.
Two bodies of masses m1 and m2 are hanging from the light and inextensible
strings, as shown in the figure. Find the tension in the strings 1, 2 and 3.

8.

Two blocks (2 kg and 1 kg) have rough surface and coefficient of friction

between them is
= 0.6 as shown in figure. The blocks are given
velocities in the directions as in figure (g=10m/s 2).Find:

9.

10.

(a) Time when relative motion between them is stopped.


(b) The common velocity of the two blocks.
(c) The displacements of 1 kg and 2 kg blocks upto that instant.
Two blocks m and Mare placed on a platform which moves up with an upward
acceleration a. Find the normal reaction between the blocks.

Calculate the value of so as to keep the smooth wedge stationary. The string
is light and inextensible which is connected to the block m as in figure.

s
11.

A 6 kg block is kept over a rough surface with coefficients of friction

k
=0.6and
=0.4as shown in figure A time varying force F=4t (F in newton
and tin second) is applied on the block. Find the relation between
acceleration of block and time (g = 10 mIs 2).

12.

13.

14.

15.

A man of mass m is standing in a lift which moves down with an upward


acceleration a. Find the pseudo force acting on the man as observed by
himself. Also find the pseudo force acting on the man if the lift falls freely.
A 6 kg block B rests as shown on the upper surface of a 15 kg wedge A.
Determine (a) the acceleration of A, (b) the acceleration of B relative to A,
immediately after the system is released from rest assuming surfaces are
smooth (g = 10 mIs2).

Two particles of masses in1 and in, are interconnected by a light inextensible
string as shown in the figures (i) and (ii). In figure

(i), free end of the string is pulled by a force F = m 3g. In figure


(ii), the free end of the string is connected to a mass m3
(a) Find the acceleration of the particles,
(b) Find the tension in the strings when either m 1 or m2 is zero.
If the cart is given an acceleration of a = 2 m/s 2 down the plane as in figure,
determine the normal force the 10 kg crate A exerts on the smooth cart.

30, g 10 m / s 2 .

Also, find the acceleration of the crate

16.

A bar of mass M is connected with one end of an inextensible and light string
that passes through three movable pulleys A, B and C fixed with the bar and

four pulleys fixed with the other end of the string connects a hanging mass
m. Find the
(a) acceleration of the bodies,
(b) tension in the string.

17.

18.

A horizontal force F acts on a point P on an inextensible light string that


passes over a light pulley which is attached with a sledge of mass M.
Neglecting friction between all contact surfaces, find the

(a) accelerations of M and m,


(b) time after which m strikes the pulley,
(c) reaction at the pulley.
A block of mass m connected with a fixed point by a light inextensible string
is kept on a smooth wedge of mass M and angle of inclination
is parallel to the incline at the time of its release, find the

19.

. If the string

(a) tension in the string,


(b) accelerations of M and m,
(c) contact forces between M and m,
(d) reaction force offered by ground on the wedge.
As in figure, an L shaped body of mass M placed on smooth horizontal
surface. The block A is connected to the body by means of a light string
passing over a smooth and light pulley. Another block B of mass m is placed
against a vertical wall of the body. What is the minimum value of the mass of

block A such that block B remains stationary relative to the wall if coefficient

of friction between the block B and the vertical wall is

20.

Two blocks of masses m and M are connected by an inextensible light string.


When a constant horizontal force F acts on the block of mass M,
find the

(a) tension in the string,


(b) acceleration of the blocks,
(c) normal reactions on the blocks by the ground. Assuming m does not lose
contact with the ground and ground is smooth.
21.
Blocks of masses m1, m2 and m3 are placed in contact on a smooth table.
If a force F is applied horizontally on m1, what is the contact force between m1
and m2?

22.

23.

24.

A body of mass 100 kg starts moving on a rough surface with uniform


velocity. If the coefficients of static and kinetic friction are 0.5 and 0.48
respectively, what is the subsequent acceleration of the body?
The coefficient of friction between A and floor is 0.4 and that between A and
B is 0.3. Find the minimum force F required to move the block away from the
wall.

What must be the minimum acceleration of A to keep bodies B and C of


equal mass stationary with respect to A. Also, find the minimum force.

25.

In figure, find the mass M of the hanging block which will prevent the smaller
block from ping over the triangular block if m 1 = 1 kg and m2 = 4 kg. All the
surfaces are frictionless an( strings and the pulleys are light.

26.

A block is placed on an inclined plane as shown in figure. What must be the


friction force between block and incline if the block is not to slide along the
incline when the incline is accelerating the right at 3m/s 2(g = 10 mIs2)?

27.

A balloon with sand weighs 1600 kg. Its lifting capacity is 1200 kg weight. It
is descending' uniform velocity. What weight of sand must be thrown away
out of the balloon so that it rises the same uniform velocity?
28.
Two persons of masses m1 and m2 are hanging by holding the inextensible
light string. Find

(a) the frictional force acting on the persons.


(b)the tensions in the segments 1 and 2

29. A force F = (10 t - t2)N acts on a body of mass 100 kg. How far will it
move, starting
from: before it starts to reverse its direction.
30.
A crate of mass m is placed on a open rear truck which moves with an
acceleration a. Find minimum value of the tension T required to slide the
crate on the open rear truck.

31.

A small cube of mass m is imparted with a velocity v 1 on a long horizontal


plate at t = 0. The horizontal plate of mass M moves with a constant
acceleration a. If at t = 0, the velocity of the horizontal plate is v 2 (> v1 ),
assuming u as the coefficient of friction between the cube and plate, find the
(a) variation of x as the function of time,

a s g ).
force acting on the plate in horizontal (assume
(b)
(c) variation of velocity of the cube as the function of time.

32.

A horizontal force F acts on a block of mass m 1, which is placed on a block of


mass m2; m2 is placed on another block of mass m3 which is placed on a
smooth horizontal floor. Let us now pull the blocks with the forces of
magnitudes 18 N, 100 N and 15 N as shown in the figure. If the coefficients of

s
static and kinetic friction between all contact surfaces are
= 0.2 respectively, find the
(a) friction at each surface,
(b) acceleration of the blocks. Assume, m1 =M2 = m3 = 10 kg.

(c)

Answer Keys
1.(b)
2. (a)(c)
3. (b)
4.(a)(c)

SECTION B
32. (c)
63. (b)(d)
33. (c)
64. (b)
34. (c)
65. (a)(b)(c) (d)
35. (a)
66. (c)

k
= 0.3 and

5.(a)(b)
6.(a)
7. (b)(d)
8. (a)
9. (c)
10. (a)
11.(a)(b)(c)
12.(a)(b)(c)
13.(a)(d)
14. (c)
15. (d)
16. (a)
17.(d)
18.(a)
19.(c)
20.(a)
21.(c)
22.(b)
23.(a)
24.(c)
25. (b)
26. (c)
27. (b)
28. (b)(d)
29. (d)
30.(a)(b)(d)
31.(c)

C-1.

1.

C-2.

I.

C-3.

1.

C-4.

C-5.

1.

C-6.

C-7.

1.

C-8.

1.

(c)
37.
38.
39.
40.

36. (b)
67.(a)(b)(d)
(a)(d)
68. (a)(c)
(d)
69. (a)(b)(c)
(a)
70. (a)(b)(c)
(b)
71. (b)
41.(b)
72.(c)
42. (c)
73. (b)
43. (c)
74. (d)
44. (a)
75. (b)(c) (d)
45. (d)
76.(a)(b)(d)
46.(a)(b)(d) 77. (c)
47.(a)(c)(d) 78.(a)
48. (a)(b)(c)(d)
79. (d)
49. (a)(b(d) 80.(c)
50. (a)(b)(c)(d)
81.(b)
51.(a)(c)
82. (b)
52. (a)(b)(c) 83. (b)
53. (a)(c)
83. (b)
54. (b)(c)
84. (d)
55. (a)(b)
85. (b)(d)
56. (d)
86. (a)(c) (d)
57. (a)
87(a)
58.(d)
88.(c)
59.(b)
89. (b)
60. (c)
90.(c)
61.(c)
91.(b)
62. (d)
92.(d)
93.(b)
SECTION C
(d)
(d)
(d)

2.
2.
2.

(d)

(d)

2.

(c)

3.

(d)

3.

3.

(c)
2

c)
(b)

3.

.(d)

(c)

(b)

2.
(a)

3.

'(a)
(a)
(a)
(c)
(c)

(a)

(d)

3.
3

3.
(a)

(b)

(d)
4.

(b)

(a)

C-9.

1.

C10. 1.
C11. 1.
C-12. 1.
C-13. 1
C-14. 1

C-16. 1.
C-17. 1.
C-18. 1.

C-20. 1
C-21. 1
C-22. 1

C-25. 1

3. (a)
4. (a)
5. (a

6. (a)

2.

(C)

2.

(d)

2.

(b)

-(a)

3.

(d)

(b)

(c)

(d)

2.

(a)

2.

(a)

2.

(d)

2.

(d)

(b

2.

(d

2.

4(a)

2.
(b)

(c)

2.

(p) (q)
(p) (r) (s)

(b)

(p) (r)

(p) (q) (r)

(b)

(s)

(b)
(b)

(b)

3.

3.

(c)
(s)

(p) (s)

3.

(c)
(p) (s)

(p) (r) (s)

(c)

(r)

(c)

(d)

(a)

(a)

(a)

(b)

(a)

(c)

3.

(a)

(p) (r)

(b)

(a)

(d)

(b)

(b)

3.

(a)

(b)

3. -

(b)

.(d)

(c)

(a)

SECTION D
(p) (q) (s)

(b)

(a)

(b)

3.

(b)

(c)

3.

(a)

2.

(c)

(b)

2.

(b)

(c)

3.

(b)

(a)

(d)

3. -

(a)

3.

(c)

(b)

2.

(d)

2. (a)

(c)

C-26. 1.

1.(a)

C-23. 1.
C-24. 1

C-15. 1.

C-19. 1

3.

(a)
(b)

(c)

(p) (q)
(c)

(d)
(s)

(q) (s)
(c)

(d)
(p)

(r)

(d)

(q)

(d)

(q) (s)
(d)

(p) (s)

(q) (s)
(d)
(r)
(p)

(p) (r)

7. (a)
8. (a)
9. (a)

(s)

(b)

(r) (t)

(a)

(b)

(q)
(b)

3 g

25

(t)

(c)
(p)

(r)
(c)

(d)
(q)

(p)
(d)

(c)
(q)
(d)
(t)
SECTION E
1. (a) 2. (c) 3. (b) 4. (d) 5. (a) 6. (c)
7. (d) 8. (a) 9. (b) 10.(c) 11. (d)
12. (a)
13. (d)
14. (c)15. (d)
16. (b)
17. (d)
18. (c)
19. (b)
20. (d)
21. (a)
22. (a)
23. (b)
24. (a)
25. (a)
SECTION F

1.

(r)

down the plane

2 g

2.
3.
4.

(b)
down the plane
(c) zero
(a) 4.9N
(b) 24.5 N
(c) 0.2
v = 5.5 m/sec.
(a) 1 sec.
(b) 6 m/sec.
(c) 4 m, 7 m (Both towards left)

T [ M (l x]

F
M l

5.

a
6.

(a)

(b)

F
g
M m

m
F

T M x
l M m

m12 m22 g

7.

2 m, g 3 m1 g

8.
9.

7
s
6

(a)
(b)
4m/sec.
(c) 12.83 rn (towards left), 0.58 m (towards left)
N=m(g+a)

(q) (s)

10.

11.

mg sin

cos

t 4 for t 9sec
3

a 0 fort 9sec
FPseudo ma()
Fpseudo mg ()

12.
13.

(a) 6.36 m/sec2

(b) 5,5 m/sec2

2m2 m1
g
m1

Fig ( i )a1

m2 m3
gand same as for
m2

a2
14.
15.

16.

fig. (ii), T = m3g


90 N, 1 m/sec2

7m M g
M 49m

(a)

a1
17.

(a)

54 Mmg
7( M 49m)
(b)

2F
F
, a2
M
m
Mml
M 2m F

(b)
(c) N = 2F
18.

(a) T= mg sin

aM
(b)

N
(c)

mA
19.

mgsin cos
mgsin 2 cos
,
a

m
M m sin 2
M m sin 2

Mmgcos
M m sin 2

N'
(d)

( M m) m g
M m sin 2

M m
and 1
1

T
20.

Fm
( M m) cos

(a)

a
(b)
(c)

F
M m

Fm
tan
M m
Fm tan
Mg
M m

N m mg
N2m

m2 m3
m1 m2 m3
21.
22.
23.

0.196 rn/sec2
18.62 N

g (1 ) (mA mB mC ) g(1 )
,
( 1)
( 1)
24.
25.

26.
27.
28.
29.
31.(a)

6.83 kg

9
mg
25
800 kg
(a) fs1 = mg, fs2 = m2g
(b) T1 = (m1 + m2)g, T1 = m2g
14.06m

1
x1 v1t ( g)t 2
2
1
x2 v2t at 2
2
r
r
F mgi and F mai for a s g
32.

(b)
(a)f12=20N, f23 = 4 0N, f3G = 25 N
(b) a1 = 0.162 m/s2, a2 = 4.11 m/s2, a3 = 0

HINTS/SOLUTIONS 2.

SECTION B
(a). (C): F = 4m1 and F = 6m2

m1 3

m2 2

F1 F
m1 m2
4 6
12
F (m1 m2 )
5
12
2.4m / s 2
5
Now,

4.

(a), (C):

m1 m2 given
T2

g
m2

acc n ofm1 oandacc n ofm2

6
g 15m / s 2
4

6.

(a):10g-4g=4a

7.

(b), (d): F1 = constant and F2

F1 kv
r r
mdv
r
also( F1 F2 ) ma F1 kv
dt
F kv
a 1
decrease from its initial value to zero
m

dv
dt
Now

F1 kv
n

In( F1 kv ) 1
t c
k
m
In( F )
Q att 0, v 0
c
k
In( F1 kv ) t InF1


k
m
k
1
t
[ In( F1 kv) InF1 ]
k
m

1 F kv
t
i.e, In 1

k
F1
m

F1 kv
kt F1 kv
e kt

F
m
F

1
1

In
v

F1 kt
(te )
k

i.e., velocity will increase from to a constant value.


10.

(a): 7g=N+Tcos450

N 70 30 2
andFs 30 2

1
30
2

1
30
2

F N 2 FS2 402 302 50 N

F 2t 2
11.

(a), (b), (c):

dv

mdv
2t 2
dt

1 2
t3
t
dt

c
2
15

i.e., velocity varies with time

at .t=0, v=0 .c=0

125 25
m/s
15
3
27
andvatt 3sec
1.8m / s
15

vatt 5sec

12.

(a),(b),(c): F=at-bt2

For F to be maximum

a 2bt 0i.e, t
Fatt a /2b

a
2b

aa
a2
a2

b 2
2b
4b
4b

Impulse

2 a /b
max

at 2 bt 3
Fdt

3 0
2

a
a
a
2

2
8b 24b 12b 2

14.

(c)

1 2
kx mgx
2
kx 2mg
kx Mg
From Eqs. (1) and (2),
2mg=Mg

m
18.

dF
0
dt

M
2

(a):F=(m1+m2)a

2a/b

F=R=m1a
and R'=m2a

m2 a m1 g

F
m1 gFromEq (1)
m

m
1
2
(m m2 ) m1 g
F 1
m2
m2

19.

(C): F=(m1+m2)a

also, m1a m2 g
From Eqs. (1) and (2),

F
m1 g
m1 m2
(m m2 ) g
F 1

m1

F
20.

(a):

k
i
x2

and at t=0 ,x=a

mdv k
dv dx k
2 m 2
dt
x
dt dt x
k dx
m( v)dx
x x2

x
v 2
k
dx

2

2 0 m x a x

integrating, we get

v 2 k 1

2 m x a
2k x a
v2

m ax

22.

2k x a

m ax

(b):F - 2t=ma

(m m2 ) g
F 2t
2t

2 1
(m1 m2 ) (m1 m2 )
(m1 m2 )

Integrating, we get v =

24.

Similarly, V=
(c) mg= T

T Mg sin 30
Mg
2
M

2
mg

29.

(d):

1
2m

1
m

t2 -2gt+c

t2 - gt + c
T= 2T
T = T

Mg
2

For block to does not slip on the car surface,

g sin cos a g sin cos


4 3
2

a g

2 3
3 2
g
2g
a
3
3

31.

(C): acceleration = g sin

+ a cos

1
l (acc n )t 2
2
2l
t
acc n
t

2l
g sin a cos
From (1)

34.

(c):

1 2
2d
at d t1
a gsin 45
2
a

2l
2 2d

0
g sin 45
g

when friction accounts,

a = g sin 45- g cos 45


then t2 in this case,

(1 )

g
2

2d

t2

g
(1 )
2

2 2d
g (1 )

Given t2 = nt1

2d 2 2d
1
; g(1 ) g n 2 1 2
n
1
k 1 2
n

35.

(a): 2k = 2mg :. x =

Mg
k

New after breaking of one spring,


mg - T = ma and T+mg-kr=ma mg-ma+mg-mg=ma

mg=2ma

g
2
mg
T
2

(net change in the string tension.)

x1 x12 h 2 y1
38.

(d):

=constant differentiating both sides

dx
1
dx dy

2 x1 1 1 0
dt 2 x12 h 2
dt
dt

`
u2+-sin

u2+u1 =0

u2(1+sin

)=u1

40.

9
9
9 4
tan a

12
0
a
tan 37
a

(b):
Also N sin 370 = ma

10 12 5
200 N
3

41. (B)

2T cos F .........(1)
alsoT sin ma....(2) From(1)and (2)
F
F
ma
sin tan
2 cos
2

F
x
x
.
tan

2 a2 x2
a 2 x 2
F
x
a

2
2m
a x2
42.

(C): For slipping


F=(M+m)a (1)

Fsmax ma

M m

mg m

F ( M m) g
kt ( M m) g
t
43.

( M m) g
( M m) g
t
k
k

(C): FFkT=ma (1)


and also, TFk =ma

From (1) and (2),

a
48.

(2)

F - 2Fk = 2ma

F 2 mg
F

g
2m
2m

(a), (b), (c), (d)

T2 sin 2 mg
(1)

T2 sin 2 mg

2
tan

again,

T1 sin

1
T1 sin

=mg

From (1)

1
and

T1cos

= T2 COS

1
T1cos

tan 1

= 2mg

1
2

+mg

2
, =T sin

T2 2mgandT1 5mg
56.

i.e
(d):T= mg +ma

T1=m(10+1)=11m (a=1m/s2,010s)
T2 =m(10+0)=10m
(:a=0,10-20s)
T3 =m(10-2)=8m (a=-2,20-25s)
T1:T2:T3 =11:10:8

2t
2 2.5

0.1(att 2.5)
mg 5 100

57.

(block just to slide )

k
After 3 sec, F-

2 3 k .5 10

60.

N=ma

5 10
10

8
0.08
100

(c): For no slipping


F = 3ma

and ma +

mg =

64.

3F
3

2F
3mg

T = ma
Mg - T = Ma

mg = F

Mg
( M m)

Mg=(M+m)a

Mg
g
( M m)
T=ma=

T1

M
M

g
2 ( M m)

Now a2 =

M
m

T2

M M
g
2 m

1
1
:
M : ( M m)
( M m) M
65.

T1: T2=
(a), (b), (c), (d)

Fs
=N'sin

= Mg cos

x sin

mg
2

sin 2

2 900

0
45

Fs
at this angle

max
s

maximum

mg
2

66.

(C): F = -

mg

=> a = -

dv ug dt
g dt

v=
->area under
velocity after 4 s,

-t curve

0.4 1 0.6
0.7 1
2
10 6m / s 2
10 2 10
10 2

20.5
70.

(a), (b), (c)

72.

m1gsin30-TT-m2g=m2a
T =20 N
(c):

Fcos

F(cos

N = m1a (N= mgcos 30)

(mg-Fsin
sin

)=

)+umg

g(M+m)

g ( M m)
(cos cos )

For F to be minimum,
i.e., -sin

= tan

cos

i.e.,
=tan-1(
74. (d)

dF
0
d
=0

a0 = 2 m/s2

From figure, T -

81.

0.3
50 10 50 4 350 N
10

(b):

tan

mAg = mAa

Mgsin

mgcos

=30O

1
h=R-Rcos300=R

Vl1
82.

(b):

l1 l
l1 l2

F1
k

F1
k
F1 F2
k

F2I1 F1l2=( F2- F1)l

1
3

F2l1 F1l2
F2 F1

Vl2

F2
k

l2 l
k

F2
k

F1 F2
l1 l2

F1 F2
k

Vl1 Vl2

Now, l3=l+ l1 +

l2

F2l1 F1l2 F1 F2

F2 F1
F1 F2

l1 l2
x(

1
( F2l1 F1l2 )
( F2 F1 )

( F1l1 F1l1 F2l2 F1l2 )


1
( F2l2 F1l1 )
( F2 F1 )
83.

(b):mg cos
Mg sin

tan

=N

=F

tan

x2=
2

x (1+
84.

2 2

r -

)=

x
R2 x2

so that

x2
2 2

r :. h=rx

(d): mgl(cos

V2 =2gl(1 cos

And T - mg =

)=

1
2

mv
l

T = mg+2mg(1cos
:.mg(1+2-2cos

mV2

0)=

x4mg

3 2 cos 0

l'
87.

(a):

T1

2 1
R
4
4 2

g
2

Now,

dT dmcos g

T Rg (sin )
Rg
Tthread
88.

R 2

/2
0

Rg (1 0)

2
R

g g2
1 2

g
2

Fs1
(C):

=0.5x2x10=10N

Fs2
=5N
Let acc of whole system be a
n

For C, then, ma =

For B, a =

5
4

m/s2

max
s1

+m a=5+3x

max

5
m / s2
4

max
s

5
4

=8.75m/s2

max

m'a F s

For this acc. block B does not slip

93.

max

5
4

95
4

=(2+3+4) =
=11.25N
(b): Tension at the ends of the chain

mg
sin
2

and tension at the mid-point

mg
mg

cot

2
2 tan

C
SECTION C
C-2

1.

(d): Fs1>Fs2

1
i.e.,

2
mg>

2mg

2
>2

1
2.
3.

(c):

>0

max

max

s1

(a):

=06x5x10=30N
s2

and
.'.

=0.2xl0x10=20N

max

=20N

C-3

By constraint equation,
aA + 2aB + 2ac =0
a+4a1 =0
2mg-2T=2ma
And 2T-mg=ma1
Mg=2ma+ ma1

a1
1.

2.
3.
C-5

g
9

(d)

(d):T=mgma=mgm
(a): mg

4
9

g=

5mg
9

s1

s3

s2

=45

=160

= 240
50 kg block will slip,

3
90a =80+250+200-450

3
-240

530 295 3
2
90

and 40 kg block also slips.

1.

(d): Normal reaction=

120 3 10
2

3
=600

2.
3.
C-6
1.
2.

(c): Normal reaction =


(c)

80 3 6
2

(c)
(a):F1 =(mA+mB)gsin300

=(40+15)(10)

1

2

=275N

3
= 400

Now, when F = 2F1 = 550 N


Acceleration of block system upwards

550 275
5m / s 2
55

To block B, aH is provided by friction.


Hence

mB aH=

N=

mB(g+aV)

aH
5cos 300
3

0
g av 10 5sin 30
5

3. (d): N = mB(g + av) = 15(10 + 5 sin 300)


=187.5N
4.
(d): When F= 0, the block system will move down the plane with acceleration
5 m/s2 and there will be no slipping between A and B. Therefore, acceleration
of B relative to ground is also 5 m/s 2
C-7

F F
a=

30
35

max
s

= 0.85 m/s2

250 40
10

a'A =
=21m/s2
and aB ==1.6m/s2
1. (C); 2. (d); 3. (a); 4. (b)
C-8

T=m1a1=1.3a1

(1)

And T

3
5

4
5

3
5

T+ N=m3a3 =3,45 a3.. (2)

4
5

and m2g+ m2a3 T=m2(a1 +a3)


=>9+1.2a2T=1.5(a1+a3)

4
5

.(3)

and m2g=N+m2a3 sin 37


12=N+0.9a3
..(4)
Solving these four equations we can find T, a 1 and a3
1. (b); 2. (a); 3. (b)
C-9

m=5 kg, m=10kg


F=2T1 andT1=2T
F=4T
1. (c): When F= 100
T=25N
m and M will not accelerate
i.e., accn of both m and M= 0
2.
(a):ma1=75-50

25
5

3.

a1 =
=5 m/s2
a2 = 0
(d):ma1=125-50

a1 =

a2 =
C-10

75
5

=15m/s2 and

100 75 25

2.5m / s 2
10
10

M1=5kg
M2=4kg
M3=2.5kg

1.
2.

3.

=0.50
From figure,
(c)

(d): T+mgsin37

3
5

(b):T+30x -0.5x50x
T + 10= 5a1

a1

mgcos37=ma1

4
5

=5a1

T
2
5

(1)
again T-0.5x40=4a2 T-20=4a2
T-20=4a2
(2)

a1 a2

25-2T=2.5
50-4T=2.5(a1+a2)(3)

T 10
5

= a1 and

a1 a2
And

T 20
4

= a2

T 10 T 20

5
4

9T 40 100
20
a1 a2 9T 60

2
20 2

25-2T=2.5

9T 60

40

From(3)

T
1150=102.5T

a1
a2

11500
11
1025

21
4m / s 2
5
11 20 9
m / s2
4
4

C-11

(l-x) 2=y2+c2

dy
dx
2(l x ) 2 y.
dt
dt

dx

dt

y
dy

1 x dt

Apply energy conservation:

(2m) (10) (3) = (2)(m)(10)(1) +

+2x

1
2

5
17
v=10

(2m)v2

x m x (0.6v)2

5
17
m/s and 0.6v=6

1
2

m/s

1. (d); 2. (b); 3. (b) C-16


C-16

m1
sin
m2
And Tm2gsin

And m1gm2gsin
= (m1 + m2)a

m1
m2
1.

2.
3.
C-18

(a):

(a):
(c):

m2gcos

=m2a

m2gcos

= sin -> no motion takes place

m1
m2
m1 >If

> (sin

.m2>1=sin +

cos

cos

50-5=10a
a=4.5m/s2 down the inkling
T=5N, ap= 2 = 2.25 m/s2 towards right
1. (d); 2. (a); 3. (b)
C-19
Fsin

Fcos

mg
(sin cos )

= mg

-> F cos
F cos

(mg +F sin )

Fsin

mg

mg
cos sin
F=

mg sin
sin( )
1.
2.

angle of friction =
(b)
(a)

3.
(a) <a
C-20
From figure,
mg sin

a = g(sin
= g(sin
a=g sin

for v

max

mg cos

=ma

k cos )

sin x)
(lx)

=g sin

(I-x) dt

v dv gsin l x dx
2

v max
g sin

2
2

=>
=>
and for Xmax, v = 0

v max g sin

v2
2

=gsin x x+
x=2m

1
2

x2 x gsin

Fs =kx x mg cos =x x mg sin

= 2mg sin
1. (b); 2. (b); 3. (a); 4. (b)
C-21

Applying constraint eqn in string 1


x1 + 2xp + x1 x2 = constant
a1 +2ap +a1 a2 =0
2a1+2apa2 =0
and for string 2,
x2xp=0
=> a2=a1
and 2TT=ma2
T=ma2
:.F-2T=ma,

=> F - 2ma2 = ma1 F = 2ma2 +

2F
5m

a2 =
(left) and T=
1. (b); 2. (b); 3. (a)
C-23
1. (d); 2.(c); 3.(d)
If elevator

2F
5

with constant velocity

mam , Incline
= ma0 sin + mgsin

2l

g sin cos

Ttoreachthebotton

1
2
=

(g+a0) sin

ma2
2

xcos2

1/ 2

= gsin

(g + a0)

2l
( g a0 ) sin cos
T=

C-26
1. (a): F=m(g+a)

2. (d): Fa = (M + m) (g + a)
3. (C): Rr =(M+m)(ga)
SECTION D
5.(a)

(p), (q), (r); (b)

(p), (r), (s) (c)

();(d)

()

When F 0
Then T 0
Fn 4 N and Fs2 0

b When F

4N

T 0
Fs1 4 and Fs2 4

F 24 N
then, F T 8 2
16 T 2
.. T 14 N
T F Fs 2 N
d F 10 N
6. (a)

(s); (b)

(r); (c)

(q); (d)

(p) From figure,

T m1 a g m1a1
m2 a g T m2 a1

1
2

From 1 and 2 ,

m2 a g m1 a g m1 m2 a1
a1

(m 2 m1 )(a g)
(m1 m2 )

distance covered in elevator frame

1 (m 2 m1 )(a g)
2

t
2
(m1 m2 )
a1 a

m 2 a m1 a gm 2 gm1 am1 m2 a
(m1 m2 )

m2 m1
g
m1 m2

8.

(a)
(c)

(r), (t); (b)


(q); (d)

T mg ma

(p)

(p), (s)

And 4g-20=4a

a=

20
4

9. (a)

=5m/s2

(r); (b)

(t); (c)

(q); (d) mg sin 30-42 = ma

1
36 10 42 6a

12 6a

: a = -2

R 42 17 25

SECTION E
1. (a): For forming an action reaction pair, cause and nature of force should be
same. both statements are correct and also statement (II) correct explanation for
statement (I).

6. (C): statement (II)


false
kinetic friction opposes relative motion of th body.
17. (d): If elevator moves downward with some acc n. then it will happen so as in
statement (I).
SECTION F

mgsin macos ma '


1. (a)

a'

3 g 12 g 3 g

5
25 25

3g g 4
2g
g
5 4 5
5

3g 3g 4
g 0
5
4 5

(b)

(c)

3.

Fs max 2 10 0.1 2 N
0.5t 2
20
t
4
15
i.e., for 4 s, both will move together,

t
t2
a v
12
24
Now, vatt 4 s

16 2
m / s2
24 3

again after 4 s,
For 2 kg block

2a

0.5
t 2; t 4 s
102
v

10

10

dv 1
1
t 1 dv tdt dt
dt 4
4 t 4
v0
t 4
10

1 t 2
v v0 (t)10
4
4 2
2 1 42
v 6asv0 2 / 3
3 4 2
31
v 5m / s
6

4.

Fk 1 x 10 x 0.4 4 N
For 1 kg block , v 2 4t
Now for 2kg block , v 8 2t
2 4t 8 2t at this time relative motion stop
6t 6

vcm 8 2 6m / s
and displacement of 1 kg block

1
2t 4t 2
2
2 2 4m
displacement of 2 kg block = 8t t2 = 8-1

5. F=(M+m)a :. a=

(m= l)

F
M m

( =m/l)

F
M l

andT ( M m' )a m (l x)
( M l x )
6.

F
(M l )

F
x
F 1

(M l )
(M l )

F ( M m) g ( M m)a
F ( M m) g
a
( M m)
T ( M m' ) g ( M m' ) a
mx
F
M mx


T M
g
g
1 M m
1

F
mx
M mx


mx

g M M g

1 (M m)

1
l
F
M mx

1 (M m)

8.
For 2 kg block, V = 3 + 6t
and for1 kg block, v=18-12t

equating (1) and (2), t =


Vcommon =18-14= 4 m/s

7
6

:.displacement of 2 kg block

= 3t +

1
2

x 6 x t2

(1)
(2)

7
6

49
36

7
12

=-3x
+3 x =
=
=0.58m (left)
and displacement of 1 kg block

= 18t-

1
2

x 12 x t2

77
6

49
36

=18x
-6 x
= 12.84 m(left)
13. Let the acceleration of 6 kg block w.r.to wedge be an horizontally leftward and
acceleration of 15 kg block be A down the incline.
then,

r
r
r
a m a mM a M
For 6 kg block,

M (a A cos ) 0

a Acos

(1)

Mg N ' MA cos
and
Now for 15 kg block,

(2)

N ' sin mgcos MA


(3)

( Mg MAsin) sin mgsin MA


mgsin MAsin 2 Mgsin MA
Ma
Ma
Mgsin
sin2 Mgsin
cos
cos

M m gsin cos Ma Masin 2

(m M ) gsin cos
(M msin 2 )

a
(m M ) gsin
2110 1 1

2
cos (M msin ) (5 6) 13 2
4
70

6.36m / s
11
A

Now
14.

a6 kgw.r.t15kg

= 6 x 36 x

3
2

= 5.5 m/s2

(i ) F T2 T1 m2 a2
m2 a2 m3 g m2 g
a2

(m 2 m 3 ) g
m2

again, 2m2 g m1 g m1a1


(2m2 m1 )
g
m1

a1

(ii) T m 3g
=(M+l_x)x(Ml) =Fl_(Ml) 6. F(M+m)g=(M+m)a F
16.

am 7 am 0andMg T M 7a1

7T Mg Ma1
Mg
6 M

M
M
g 7 Ma1 a1
7
7
a
g (49m m) 1
7

7m M
g
49m M

a1

49m 7 m
g mg
49m M

a2 7 a1

56mMg
7 m M 49m M

49m M
7(49 m M)

Mg

F ma

17.

F
M
2 F Ma '
2F
a'
M
F 2F
1 2
Now, arel
F

m M
m M
1 1 2
(b) l F
t2
2 m M

2mMl
t

F (M 2 m)
(a) a

(c ) N 2 F

18. From Figure,


Nsin

= ma

and mgcos

N = masin

mg cos

Ma
ma sin
sin

mg cos m sin
a
sin

a M sin 2 M mg cos sin


mg cos sin
M sin 2 M
again, T Mg sin
Mmg cos
N
(M msin 2 )

Mmg cos 2
and ..N (byground )
Mg
M m sin 2
Mg ( m cos 2 M m sin 2 )
( M m sin 2 )
'

M (m M ) g
( M m sin 2 )
19.

Fsmax mg

0r ma mg
a

again, M ' g T m 'a


m' g ma ( M m ' )a

g
m' g M ' a Ma ma puta

'
M ( 1) (M m)
M '

M m
andalso 1
1

Fmin Fs1 Fs2 16 3 N 19 N


23.

F min 18.62 N take g 9.8m / s 2

fs
24. Let m1 tends to slip towards righ, then
Force equation for m1 w.r.t. the wedge.

acts towards left on m1.

m1a+ m1gT=0
(1)
Force equation for m2 w.r.t. the wedge.

T=m2g

m2a

Solving eqns. (1) and (2)

a min ( a )

m2 m1
1
g
g ( asm1 m2 )
m1 m2
1

similarly, a max

m2 m1
g
m1 m2

1
g (asm1 m2 )
1
(m m B m C ) g(1 )
and ..F min A
( 1)

(2)

25.

From Figure,

T (m1 m2 ) gtan 300


andmg T mg tan 300
mg (m1 m2 ) g tan 300 mg tan 300
m1 m2
g
3

m m2
5
5
M 1

6.84kg
1.73 1 0.73
3
mg (1 tan 300 )

26.

mg cos 370 mg sin 37 0


From figure,

4 g 3 3 4

5
5
5

40 g 3g 12 18
and Frictional force

3
3g 4
F mg m
5
10 5
9
3 6
mg
mg
25
5 25

9
0.75
20

30.

tan cos ma (mg T sin )


Tcos Tsin mg ma

T(cos sin ) mg ma
T

m( g )
cos sin

maximum value of
(cos sin is 1 2
T min

m g a
1 2
.

31.

r
a1 g i

Since V2 > V1, kinetic friction on m is formed. Then

r
v 2 (v 2 at ) i

The velocity of the planet is


Then the relative sliding will stop.

fk
Then
will cease to exist. If
velocity.

When

. Hence,

r r
g
v1 v 2 , t
( t 0 ).
a

a s g
,after the time to the bodies move with same

1
(a ) x1 v1t ( g) t 2 , x2 v2t a s g ( g)at 2
2

ur
(b) F k mg i ( k ) tillt t 0

ur
and F ma i ( fora s g)
32.

r
v 1 (v g ) i .

s 0.3

k 0.2

m3 will not move


so consider m1 and m2 as a system,

m1 m2 a 82 0.2
a

42
20

20 10

a 2.1m / s 2

Now for m1,

m1a1 19.6 18 1.6


1.6
0.16m / s 2
10
again.. form2 ,

a1

m2 a2 100 Fs1 Fs2


a2

100 39.2 19.6 41.2

4.12m / s 2
10
10

a3 0
also, Fs1,2 20 N
andF3, ground 25 N

Fs2,3 40 N

Vous aimerez peut-être aussi